You are on page 1of 30

PART

VII Mental Disorders Due to Another Medical Condition

21 Mental Disorders Due to Another Medical Condition


BJ Beck, MSN, MD, and Kathryn J. Tompkins, MD

KEY POINTS DISORDERS


Because affective, behavioral, cognitive, and perceptual distur-
Mental disorders due to another medical condition bances are indistinguishable by etiology, every psychiatric
should be in the differential diagnosis for every evaluation should include mental disorders DTAMC in the
psychiatric evaluation. differential diagnosis. With this rationale, the individual dis-
A high degree of medical suspicion and familiarity orders are listed in DSM-5 with phenotypically similar disor-
with the categories of causative medical conditions ders. Box 21-1 lists the various disorders under the chapter
will aid in the diagnosis. titles in which they appear in DSM-5.
Psychiatric symptoms may precede, coincide with, or
lag behind the physical manifestation of a medical PSYCHIATRIC DIFFERENTIAL DIAGNOSIS
condition.
Psychiatric symptoms may resolve, lessen, or continue
Primary Mental Disorders
after appropriate treatment of the underlying medical The determination of direct physiological causality is a
cause, and they may require specific treatment. complex issue. Box 21-2 summarizes features that should raise
The course of some medical conditions may not be the clinicians suspicion of medical causation of psychiatric
alterable, but rigorous treatment of psychiatric distress symptoms. The onset of psychiatric symptoms that coincides
is part of palliation. with the onset (or increased severity) of the medical condition
is suggestive, but correlation does not prove causation. Mental
and other medical disorders may merely co-exist. The initial
presentation of a medical condition may be psychiatric (e.g.,
depression that manifests before the diagnostic awareness of
pancreatic carcinoma), or the psychiatric symptoms may be
OVERVIEW disproportionate to the medical severity (e.g., irritability in
Mental disorders due to a general medical condition (DTGMC) patients with negligible sensorimotor symptoms of multiple
were defined by the Diagnostic and Statistical Manual of Mental sclerosis [MS]). It also happens that psychiatric symptoms may
Disorders, Fourth Edition (DSM-IV) as psychiatric symptoms occur long after the onset of medical illness (e.g., psychosis
severe enough to merit treatment, and determined to be the that develops after years of epilepsy). Psychiatric improvement
direct, physiological effect of a (non-psychiatric) medical con- that coincides with treatment of the medical condition sup-
dition.1 This conceptual language substituted for previously ports a causal relationship, although psychiatric symptoms
less useful, more dichotomous terms (e.g., organic versus that do not clear with resolution of the medical condition do
functional) that minimized the psychosocial, environmental not rule out causation (e.g., depression that persists beyond
influences on physical symptoms and implied that psychiatric normalization of hypothyroidism). There are also mental dis-
symptoms were without physiological cause. The Diagnostic orders DTAMC that respond to, and require, direct treatment
and Statistical Manual of Mental Disorders, Fifth Edition (DSM- (e.g., inter-ictal depression), which should not be interpreted
5)2 attempts to further minimize this dualism, and it makes as evidence of a primary mental disorder.
the organic nature of the brain and its symptomatic disorders Psychiatric presentations uncharacteristic of primary
and responses more explicit by changing the terminology to mental disorders should raise the suspicion of a direct physi-
Due to Another Medical Condition (DTAMC).3 The mental ological effect of a medical condition. Features to consider
disorder must be deemed the direct pathophysiological con- include the age of onset (e.g., new-onset panic disorder in an
sequence of the medical condition, and not merely co-existent, elderly man), the usual time course (e.g., abrupt onset of
or an adjustment reaction to the psychosocial consequences depression), and exaggerated or unusual features of related
of the medical condition. Symptoms that are present only symptoms (e.g., severe cognitive dysfunction with otherwise
during delirium (i.e., with fluctuations in the level of con- mild depressive symptoms). On the other hand, the typical
sciousness and cognitive deficits) are not considered DTAMC. features of a psychiatric syndrome support the likelihood that
Similarly, the presence of a neurocognitive disorder (NCD) medical and mental disorders are co-morbid, but not causa-
(e.g., dementia with memory impairment, aphasia, apraxia, tive. Such typical features include a history of similar episodes
agnosia, or disturbance of executive function) takes prece- without the co-occurrence of the medical condition, as well as
dence over a diagnosis of DTAMC. Symptoms that are clearly a family history of the mental disorder.
substance-induced (e.g., alcohol intoxication or withdrawal) Scientific standards in medical literature have established
do not meet criteria for DTAMC.2 probable medical causality for certain psychiatric symptoms
205
Descargado para JORGE LUIS SALAZAR GMEZ (jrg.lusaga@gmail.com) en Instituto Technologico Estudios Superiores Monterrey - Monterrey de ClinicalKey.es por Elsevier en mayo 03,
2017. Para uso personal exclusivamente. No se permiten otros usos sin autorizacin. Copyright 2017. Elsevier Inc. Todos los derechos reservados.
206 PART VII Mental Disorders Due to Another Medical Condition

BOX 21-1 Mental Disorders DTAMC Listed by Chapter BOX 21-3 Categories for Another Medical Condition
of DSM-5
ANotherMEDICalCONDITion
SCHIZOPHRENIA SPECTRUM AND OTHER PSYCHOTIC Autoimmune
DISORDERS Nutritional deficit
Psychotic Disorder DTAMC Metabolic encephalopathy
Catatonia DTAMC Endocrine disorders
Demyelination
BIPOLAR AND RELATED DISORDERS
Immune
Bipolar and Related Disorder DTAMC Convulsions
DEPRESSIVE DISORDERS Cerebrovascular disease
Depressive Disorder DTAMC Offensive toxins
Neoplasm
ANXIETY DISORDERS
Degeneration
Anxiety Disorder DTAMC Infection
OBSESSIVE-COMPULSIVE AND RELATED DISORDERS Trauma
Obsessive-Compulsive and Related Disorder DTAMC
NEUROCOGNITIVE DISORDERS
Delirium DTAMC clinicians should think in broad terms, and they should ask
Major Neurocognitive Disorder DTAMC specifically about over-the-counter medications, herbal
Mild Neurocognitive Disorder DTAMC (natural or dietary) supplements, prescription medications
PERSONALITY DISORDERS (not necessarily prescribed for the patient), and alcohol and
Personality Change DTAMC recreational (i.e., illegal) drugs. This detail should be routine
in every evaluation, with the more selective use of blood or
OTHER MENTAL DISORDERS urine toxicology, as indicated. Symptoms may derive from
Other Specified Mental Disorders DTAMC substance use, intoxication, or withdrawal, and such mental
Other Unspecified Mental Disorders DTAMC symptoms may persist for several weeks after the patients last
use of the substance.4 Substance-induced mental symptoms
DTAMC, Due to another medical condition.
are not necessarily evidence of misuse or abuse. Some medica-
tions may cause symptoms when used in therapeutic doses.
When such symptoms resemble concomitant disease symp-
BOX 21-2 Features Suggestive of Physiological toms (e.g., when a patient with systemic lupus erythematosus
Causation of Psychiatric Symptoms [SLE] develops mood lability and is on steroids), it is possible
that the symptoms are both substance-induced and DTAMC,
Onset of psychiatric symptoms that coincides with onset, or in which case the clinician should code both conditions.
increased severity, of a medical condition The unspecified categories in DSM-5 are reserved for
Psychiatric symptoms that improve with treatment of medical symptoms characteristic of the given mental disorder (e.g.,
condition unspecified depressive disorder, unspecified neurocognitive
Features of syndrome uncharacteristic of primary mental disorder) that cause clinically-significant distress or impair-
disorders ment in social, occupational or other important areas of func-
Pathophysiological explanations for psychiatric symptoms that tioning predominate but do not meet the full criteria for any
are based on the medical condition of the disorders in the identified diagnostic class,2 and where
Rigorous scientific literature supports medical causality for there is etiological uncertainty or insufficient information to
psychiatric symptoms that are more prevalent in certain make a more specific diagnosis.
medical conditions than in appropriate control groups
Historically-accepted connections (e.g., case reports and Another Medical Condition
small case series)
ANotherMEDICalCONDITion (Box 21-3) is a mnemonic for
the wide range of medical conditions that may result in psy-
chiatric syndromes.
that are more prevalent in certain medical conditions than the
base rate experienced by an appropriate control group. There
may also be a pathophysiological explanation for the psychi- Infectious Diseases
atric symptoms, based on the medical disorder, metabolic
The increased prevalence of immune suppression (e.g., from
perturbation, or location of brain pathology (e.g., disinhibi-
acquired immunodeficiency syndrome [AIDS], or from thera-
tion or decreased executive function with frontal lobe damage).
peutic suppression in cancer treatment or organ transplanta-
Such connections should be considered suggestive, but not
tion) has been associated with a concomitant increase in the
definitive. Every patients symptoms should be individually
chronic meningitides and other central nervous system (CNS)
scrutinized. In the absence of large studies or historically-
infections.
accepted connections, case studies or small series may support
a causal relationship. While mildly helpful, these less stringent
reports should be met with some skepticism.
Herpes Simplex Virus (Figure 21-15)
Herpes simplex virus (HSV) is the most frequent etiology of
focal encephalopathy, and may cause either simple or complex
Substance-induced Disorders partial seizures.6 With a predilection for the temporal and
Many types of substances have the potential for use, misuse, inferomedial frontal lobes, as illustrated in Figure 21-2,7 HSV
or abuse. In considering the possible role of substances, is well known to cause gustatory or olfactory hallucinations,

Descargado para JORGE LUIS SALAZAR GMEZ (jrg.lusaga@gmail.com) en Instituto Technologico Estudios Superiores Monterrey - Monterrey de ClinicalKey.es por Elsevier en mayo 03,
2017. Para uso personal exclusivamente. No se permiten otros usos sin autorizacin. Copyright 2017. Elsevier Inc. Todos los derechos reservados.
Mental Disorders Due to Another Medical Condition 207

21

Figure 21-1. Electrograph of herpes simplex virus. (Courtesy of the Center for Disease Control and Prevention/E.L. Palmer.)

or anosmia (loss of the sense of smell).8 This concentration


in limbic structures may also explain the personality change,
bizarre behavior, and psychotic symptoms that some affected
patients exhibit. Such personality changes, cognitive difficul-
ties, and affective lability may be persistent.9 HSV-1 is respon-
sible for most non-neonatal cases of HSV encephalitis, which
does not appear to be more prevalent in the immunosup-
pressed population.10 However, HSV-2, which more commonly
causes aseptic meningitis in adults, can cause encephalitis
during the course of disseminated disease in immunocompro-
mised adults.9

Human Immunodeficiency Virus (Figure 21-311,12)


Patients with human immunodeficiency virus (HIV) infection
experience a broad range of neuropsychiatric symptoms, col-
lectively referred to as HIV-associated neurocognitive disor-
ders (HAND).13 For descriptive purposes, these disorders have
been further broken down into three recognizable conditions:
HIV-associated dementia (HAD); HIV-associated mild neuro-
cognitive disorder (MND); and asymptomatic neurocognitive
impairment (ANI).13 Early cognitive and motor deficits involve
attention, concentration, visuospatial performance, fine motor
control, coordination, and speed.14 The subcortical type of
Figure 21-2. MRI of herpes simplex virus (HSV) encephalitis. Bright dementia, characterized by short-term memory problems,
signal corresponds to active viral leptomeningeal and brain tissue word-finding difficulty, and executive dysfunction, is now rare
infection of the patients right medial temporal lobe. The temporal horn (2%) in the US.15 Problems with sleep16 and anxiety17,18 are
of the lateral ventricle is obliterated by hippocampal swelling. HSV prevalent in the HIV-infected population. This is thought to
tends to attack limbic structures responsible for the integration of be the result of wide-spread antiretroviral therapy, although
emotion, memory, and complex behavior. (From www.med.harvard some antiretrovirals have themselves been implicated as
.edu/AANLIB/cases/case25/mr1/012.html.) neurotoxins (e.g., efavirenz).14 There may also be depressed
mood19 and an array of associated symptoms that resemble
the neurovegetative symptoms of depression (e.g., apathy,
fatigue, social withdrawal, and a lack of motivation or spon-
taneity). Mania and hypomania occur, though less frequently.20
New-onset psychosis is uncommon and generally seen only

Descargado para JORGE LUIS SALAZAR GMEZ (jrg.lusaga@gmail.com) en Instituto Technologico Estudios Superiores Monterrey - Monterrey de ClinicalKey.es por Elsevier en mayo 03,
2017. Para uso personal exclusivamente. No se permiten otros usos sin autorizacin. Copyright 2017. Elsevier Inc. Todos los derechos reservados.
208 PART VII Mental Disorders Due to Another Medical Condition

gp120 docking
glycoprotein
gp41 transmembrane
Lipid membrane glycoprotein

Viral RNA
Integrase

Matrix

Capsid

Reverse transcriptase
A B
Figure 21-3. Human immunodeficiency virus (HIV). (A) Micrograph of HIV. (B) Schematic representation of the virus with various components
labeled. (A, Courtesy of the Center for Disease Control and Prevention/Dr. Edwin P. Ewing, Jr. B, From the National Institute of Allergy and Infec-
tious Diseases, http://www.niaid.nih.gov/topics/HIVAIDS/Understanding/Biology/pages/structure.aspx.)

in advanced stages of disease,21 at which point patients may


also develop seizures, agitation, severe disinhibition, and
mutism.
The causes of HIV-related neuropsychiatric symptoms are
as varied as the manifestations. Direct CNS infection, meta-
bolic perturbations, endocrine abnormalities, malignancies,
opportunistic infections, and the side effects of medications
may all contribute. A consistent correlation has not been
shown between the presence, severity, or location of CNS
pathology and symptomatology.22 New onset and a CD4
count of less than 600 increase the likelihood that psychiatric
symptoms may be due to HIV infection (Figure 21-4),23 or to
some other (HIV-related) cause (i.e., not due to a primary
mental disorder).24

Rabies
Generally transmitted by the infected saliva of an animal bite,
rabies is a viral infection of the CNS in mammals. Human
cases in the US are now so uncommon (one to two fatal cases
per year since 198025) that most occur after domesticated
animal bites received during travel outside of this country. By
the 1970s, endemic rabies sources in the US moved from Figure 21-4. HIV infecting a cell. (From Jouvenet N, Neil SJ, Bess C,
domesticated animals (mostly dogs) to wild animals because etal. Plasma membrane is the site of productive HIV-1 particle assem-
of vaccination programs begun in the 1950s. While major wild bly. PLoS Biol 4(12):e435, 2006.)
reservoirs in the US are raccoons, foxes, skunks, and bats,
variant bat rabies forms are now responsible for most fatal,
indigenous human rabies cases.26 This is significant because
the variant forms may have a different course, require minimal defenses are the likely determinants of the delay from contact
inoculum, and cause infection in non-neural tissues. This to onset of symptoms, as the virus travels along peripheral
results in a less classic, and thus unrecognized, presentation. nerves centripetally to the CNS.29 This variable time course
In 2004, this difficulty in pre-morbid diagnosis became and presentation are summarized in Figure 21-5. Paresthesias
evident when four organ transplant recipients diagnosed with or fasciculations at the bite location are characteristic aspects
variant rabies encephalitis were traced back to single donor that distinguish rabies from viral syndromes with otherwise
with a history of a bat bite.27 Since more effective vaccines for similar prodromes. Physical agitation and excitation give way
post-exposure prophylaxis were introduced in 1979, there to episodic confusion, psychosis, and combativeness. These
have been no deaths following their timely use.28 episodes, possibly interspersed with lucid intervals, are the
The average incubation time (for the more classic form of harbinger of acute encephalitis, brainstem dysfunction, and
human rabies) is 4 to 8 weeks, but it is highly variable, with coma. Death generally occurs within 4 to 20 days. Autonomic
reports of periods as short as 10 days to as long as 1 year. The dysfunction, cranial nerve involvement, upper motor neuron
bite location, magnitude of the inoculum, and extent of host weakness and paralysis, and often vocal cord paralysis occur.

Descargado para JORGE LUIS SALAZAR GMEZ (jrg.lusaga@gmail.com) en Instituto Technologico Estudios Superiores Monterrey - Monterrey de ClinicalKey.es por Elsevier en mayo 03,
2017. Para uso personal exclusivamente. No se permiten otros usos sin autorizacin. Copyright 2017. Elsevier Inc. Todos los derechos reservados.
Mental Disorders Due to Another Medical Condition 209

Incubation:
asymptomatic 21
<30 days in 25% of cases
3090 days in 50%

Exposure Prodrome: early, non-specific symptoms:


headache, fever, malaise, nausea,
vomiting, anorexia
210 days
9 months 3 months
Acute neurological phase:
Furious rabies 80% Paralytic rabies 20%
Psychosis, bizarre Flaccid paralysis,
6 months behavior, biting, anxiety, paresis, plegias,
agitation, hydrophobia, ascending paralysis
autonomic dysfunction
27 days
Figure 21-5. Time course progression of Coma: Multisystem failure,
rabies infection. Bite location, magnitude of SIADH, diabetes insipidis
exposure, and degree of host defenses con- 014 days
Death: Cardiac and/or respiratory
tribute to the highly variable time course as the failure
virus travels along peripheral nerves toward Variable course,
the central nervous system. SIADH, Syndrome or
of inappropriate antidiuretic hormone. Recovery: Exceedingly rare

Approximately half of rabies-infected humans experience


the classic hydrophobia29 (i.e., violent and severely painful
spasms of the diaphragm, laryngeal, pharyngeal, and acces-
sory respiratory muscles, triggered by attempts to swallow
liquids).

Lyme Disease
Borrelia burgdorferi is the tick-borne (specifically, Ixodes
scapularisborne [Figure 21-630]) spirochete responsible for
Lyme disease, most commonly seen in the US (northeast,
upper midwest, and [to a lesser extent] Pacific coastal states31),
as well as parts of Europe. The neuropsychiatric sequelae of
Lyme disease require clinicians in all areas to have a raised
level of consciousness and suspicion because the symptoms
are non-specific, highly variable, often delayed, and recur- I. scapularis
rent.32 Even when suspected, the diagnosis is difficult because
of the confusing array of unreliable serologic tests (e.g., Lyme
enzyme-linked immunosorbent assay [ELISA], Lyme Western
blot, polymerase chain reaction [PCR] assay, or culture).33
After the offending tick bite, ticks must stay attached for
close to 36 hours to transmit the spirochete.34 Patients may
experience a mild, flu-like syndrome, and some develop a
characteristic rash (erythema migrans, shown in Figure 21-735),
most frequently (75%80% of US cases with rash) a single
lesion surrounding the bite. However, a more disseminated
rash is thought to correspond to hematogenous spread that
occurs over several days to weeks. The target sites for the spi-
Unengorged Engorged
rochete include the heart, eyes, joints, muscles, peripheral
nervous system, or CNS, where it may lie dormant for so long
I. scapularis nymph
(e.g., months to years) that memory of the initial bite has long
since faded.33,36 Rapid diagnosis and aggressive antibiotic treat- Figure 21-6. Ixodes scapularis: tick that causes Lyme disease. The
ment are the preferred course, but some patients may still deer tick life cycle, from egg to adult to egg, is 2 years; that accounts
experience the onset or recurrence of symptoms months to for the year-round presence of ticks. (Redrawn from Haynes EB,
years later.33 Controversy exists as to whether this truly consti- Piesman J: How can we prevent Lyme disease? N Engl J Med
tutes a chronic Lyme syndrome. Some researchers believe 348:24242430, 2003.)
other psychiatric conditions or chronic multi-system illness
better accounts for these prolonged symptomatic presenta-
tions,37,38 while others point to similarities in the science multiple sclerosis. Some patients go on to develop chronic
behind other chronic spirochetal syndromes (e.g. neurosyphi- encephalopathy,33 a broad scope of persistent disturbances in
lis) as evidentiary support.39 personality, behavior (e.g., disorganized, distractible, cata-
Fatigue, irritability, confusion, labile mood, and disturbed tonic, mute, or violent), cognition (e.g., short-term memory,
sleep may herald Lyme encephalitis. The much less common memory retrieval, verbal fluency, concentration, attention, ori-
presentation of Lyme encephalomyelitis may be confused with entation, and processing speed), mood (e.g., depressed, manic,

Descargado para JORGE LUIS SALAZAR GMEZ (jrg.lusaga@gmail.com) en Instituto Technologico Estudios Superiores Monterrey - Monterrey de ClinicalKey.es por Elsevier en mayo 03,
2017. Para uso personal exclusivamente. No se permiten otros usos sin autorizacin. Copyright 2017. Elsevier Inc. Todos los derechos reservados.
210 PART VII Mental Disorders Due to Another Medical Condition

or labile), and thought or perception (e.g., paranoia, halluci- and more diffuse nature of the syndrome associated with the
nations, depersonalization, hyperacusis, or photophobia). diffuse frontal and temporal lobe findings seen on imaging
Although extremely rare, more severe sequelae may include studies (Figure 21-842). Personality change may be striking,
dementia, seizures, or stroke.36 and can involve apathy, poor judgment, lack of insight, irrita-
bility, and (new onset of) poor personal hygiene and groom-
Neurosyphilis ing. Patients may also have difficulty with calculations and
short-term memory. Later signs include mood lability, delu-
Historically, less than 10% of patients with untreated syphilis
sions of grandeur, hallucinations, disorientation, and demen-
develop a symptomatic form of parenchymatous neurosyphi-
tia.43 It is during this late stage that the classic neurological
lis known as general paresis, 10 to 20 years after their initial
signs may appear (e.g., tremor, dysarthria, hyperreflexia, hypo-
infection.40 After the advent of penicillin, but before the onset
tonia, ataxia, and Argyll Robertson pupils [small, irregular,
of AIDS, neurosyphilis was even less prevalent. Now, there is
unequal pupils able to accommodate, but not react to light]).
a resurgence of a less classic form of neurosyphilis, as the use
The diagnostic paradigm has been to use non-treponemal
of antibiotics has altered the characteristic features.40 This
serologic tests for screening (e.g., the rapid plasma reagin
more subtle presentation may be especially difficult to recog-
[RPR]) confirmed by cerebrospinal fluid (CSF) with elevated
nize by the generation of clinicians raised and educated in the
protein and lymphocytes and a positive (CSF) Venereal Disease
relative absence of the disease. The signs and symptoms listed
Research Laboratory (VDRL) slide test for treponemal anti-
in Box 21-4, with the mnemonic PARESIS,41 suggest the frontal
bodies.44 However, sensitive and specific treponemal serologic
tests are becoming available for screening. Because they cannot
distinguish between recent or remote infection, or between
treated or untreated infection, they must be confirmed with
non-treponemal tests, a pattern reversal, or diagnostic para-
digm shift.45

BOX 21-4 Manifestations of General Paresis


PARESIS
Personality
Affect
Reflexes (hyperactive)
Eye (Argyll Robertson pupils)
Sensorium (illusions, delusions, hallucinations)
Intellect (decreased recent memory, orientation, calculation,
judgment, and insight)
Speech

From Lukehart SA, Holmes KK. Syphilis. In Fauci AS, Braunwald E,


Figure 21-7. Typical erythema migrans rash in a patient with Lyme Isselbacher KJ, etal., editors: Harrisons principles of internal medicine,
disease. (From DePietropaolo DL, Powers JH, Gill JM, Foy AJ. Diagno- ed 14, New York, 1998, McGraw-Hill.
sis of Lyme disease, Am Fam Physician 72:297304, 2005, Figure 2.)

A B
Figure 21-8. Non-specific MRI findings in general paresis. (A) Axial T2-weighted image shows primarily frontal atrophy and dilated third and lateral
ventricles. (B) Coronal T1-weighted image demonstrates bilateral medial temporal lobe and hippocampal atrophy. (From Kodama K, Okada S,
Komatsu N, etal. Relationship between MRI findings and prognosis for patients with general paresis, J Neuropsychiatry Clin Neurosci 12:246250,
2000, Figure 2.)

Descargado para JORGE LUIS SALAZAR GMEZ (jrg.lusaga@gmail.com) en Instituto Technologico Estudios Superiores Monterrey - Monterrey de ClinicalKey.es por Elsevier en mayo 03,
2017. Para uso personal exclusivamente. No se permiten otros usos sin autorizacin. Copyright 2017. Elsevier Inc. Todos los derechos reservados.
Mental Disorders Due to Another Medical Condition 211

TABLE 21-1 Common CSF Findings in Different Types of Meningitis


21
Pressure Predominant Glucose Protein
Condition (mm/H2O) Cells/l Cell Type (mg/dl) (mg/dl) Examples Microbiology
Normal 80200 05 Lymphs 5075 1540
Purulent 200300 1005,000 > 80% PMNs < 40 > 100 Acute bacterial, fulminant Specific pathologic bacterium
fungal, fulminant amebic identified in 60% of Gram
meningoencephalitis stains, 80% of cultures
Aseptic N or 10300 Lymphs N N or Infectious: bacterial, Viral Isolation, PCR assays
(sometimes (Reduced in (< 100) partially-treated bacterial,
some PMNs) LCM and viral, post-infectious,
mumps)) fungi, early listerial
Non-infectious: drugs, Negative findings on work-up
meningeal disease,
parameningeal disease,
neoplasm, reaction to
intrathecal injections,
vaccine reactions
Chronic N or 10500 Lymphs or TB, atypical tuberculi, Acid-fast bacillus stain,
(180300) < 40 50200 cryptococci, coccidioides, culture, PCR, India Ink,
other fungi, sarcoidosis, cryptococcal antigen,
Lyme disease, syphilis, culture
cysticercosis, tumor
Adapted from Medscape: http://emedicine.medscape.com/article/232915-differential, accessed on 9/21/2013.
CSF, Cerebrospinal fluid; Lymphs, lymphocytes; N, normal; PMNs, polymorphonuclear lymphocytes; LCM, lymphocytic choriomeningitis,
TB, tuberculosis; , increased; , greatly increased; , decreased.

Chronic Meningitis
Although Mycobacterium tuberculosis is the most common cause,
the fungal pathogens Cryptococcus and Coccidioides (and others
endemic to specific localities) may also produce this subtle,
non-specific syndrome,46 which may be ascribed to a primary,
predisposing illness, such as AIDS. The old, young, homeless,
and alcoholic are also at increased risk. The chronic menin-
gitides are treatable if recognized, but they often go undetected
because they cause minimal signs (e.g., low-grade fever) and
symptoms (e.g., mild headache), particularly in immunocom-
promised patients. The equally non-specific neuropsychiatric
manifestations include confusion and problems of behavior,
cognition, and memory. Characteristic CSF findings, summa-
rized in Table 21-1,47 include a primarily lymphocytic pleocy-
tosis with decreased glucose and elevated protein.

Chronic, Persistent Viral or Prion Diseases


These CNS infections are exceedingly rare and invariably fatal
within months to years of onset of symptoms. Even though
the psychiatric symptoms may precede the severe neurological Figure 21-9. Diffuse MRI findings in subacute sclerosing panen-
manifestations, the course is so acutely devastating as to pre- cephalitis (SSPE). T2-weighted MRI of a childs brain with SSPE
clude misattribution as a primary mental disorder. For com- demonstrates diffuse demyelination of the white matter. (From Garg
pleteness, this brief overview provides the clinician a passing RK. Subacute sclerosing panencephalitis, Postgrad Med J 78:6370,
familiarity with these historically more important entities. 2002, Figure 2.)
Subacute sclerosing panencephalitis (SSPE) is a disease pri-
marily of children and adolescents (usually less than age 11), (e.g., cortical blindness) occur in about half of cases, with
in boys about twice as often as girls, after previous measles destructive involvement of the retina, optic nerve, and the
(rubeola) infection, with early primary infection (especially visual cortex. Early imaging of the brain may be normal,48
before age 2) posing a significantly greater risk.48 Previously followed by obliterated sulci and small ventricles from cere-
thought to occur, though rarely, after measles vaccination,49 bral edema, then T2-weighted hyperintensity predominantly
the vaccination is now known to be protective, and not causa- in the occipital lobe and subcortical white matter with relative
tive, in children not already infected with the etiologic mutated frontal sparing, as seen in Figure 21-9.49 Within a few months,
measles virus (i.e., the SSPE virus).48 Since the advent of more affected children may experience myoclonic jerks, ataxia,
pervasive vaccination, the incidence of SSPE has progressively seizures, and further intellectual decline. A characteristic
decreased. The initial manifestations, which may occur 210 electroencephalograph (EEG) pattern, which corresponds to
years after the primary measles infection (mean time is 6 the myoclonic jerks,51 consists of bilateral, symmetric, high-
years50), may be mistaken for behavioral problems, with wors- voltage, synchronous bursts of polyphasic, stereotyped delta
ening school performance, distractibility, oppositional behav- waves (Figure 21-1049). By 6 months most patients are bedrid-
ior, or temper tantrums. Ocular changes and visual symptoms den, followed by death within 13 years. Despite some meager

Descargado para JORGE LUIS SALAZAR GMEZ (jrg.lusaga@gmail.com) en Instituto Technologico Estudios Superiores Monterrey - Monterrey de ClinicalKey.es por Elsevier en mayo 03,
2017. Para uso personal exclusivamente. No se permiten otros usos sin autorizacin. Copyright 2017. Elsevier Inc. Todos los derechos reservados.
212 PART VII Mental Disorders Due to Another Medical Condition

F8T4

T4T6

T6O2

FP1F7

F7T3
Figure 21-10. Characteristic EEG tracing in
T3T5
subacute sclerosing panencephalitis (SSPE). EEG
demonstrates 4- to 6-second recurrences of
T3O1 slow-wave complexes, identical in all leads, as is
common in SSPE. (From Garg RK. Subacute
sclerosing panencephalitis, Postgrad Med J
1 sec 78:6370, 2002, Figure 1.)

response to trials of intrathecal interferon-alpha and isoprino- for healthy skepticism in the dichotomous view of functional
sine, SSPE remains highly lethal, with a reported spontaneous versus organic epilepsy continues to beg the mind/brain ques-
remission rate of 5%.50 tion. This common neurological (i.e., brain) disorder is now
Creutzfeldt-Jakob disease (CJD), in contrast, is a disease attributed to indiscriminate, haphazard, electrical misfiring of
primarily of 5575 year olds. This rapidly progressive and fatal impulses in the brain cortex. However, before the use of the
prion disease is exceedingly rare, with most cases thought to EEG (Figure 21-12) could correlate these disorganized electri-
be sporadic (sCJD). Approximately 5% to 15% appear to be cal episodes with the resultant (motor, affective, behavioral,
familial, or genetic (gCJD).52 Iatrogenic, person-to-person cognitive, memory, or perceptual) phenomena, seizures were
infection (iCJD) has also occurred following therapeutic use considered to be moral, emotional, or mental (i.e., mind)
of cadaveric human growth hormone, dura mater grafts, and, infirmities.60,61 Seizure manifestations, including altered or
in less significant numbers, corneal transplantation, cadaveric loss of consciousness, correlate with the location of abnormal
gonadotropins, and surgical instrument contamination. A brain impulses, but these manifestations are the same, non-
variant form of CJD (vCJD), caused by infection with the etio- specific responses that occur in response to stimulation (in the
logical prion of bovine spongiform encephalopathy (BSE; also same anatomic area) from any input source. While generalized
known as mad cow disease), tends to affect younger adults tonic-clonic motor activity is a syndrome readily recognized
and is also implicated in person-to-person infection through as a seizure, seizure-induced fear, depression,62 anxiety, or psy-
blood product transfusion.53 The initial non-specific symp- chotic symptoms63 are indistinguishable from those of primary
toms of CJD include problems of cognition (memory or judg- mental disorders. The EEG may be suggestive of seizure, but
ment), mood (lability), perception (illusions or distortions), the clinical diagnosis of seizures cannot be ruled out by the
or sensorimotor function (ataxic gait, vertigo, or dizziness). In lack of EEG evidence64 (Figure 21-13).
vCJD, the early symptoms are more prominently psychiatric, Although a number of detailed classification systems exist,
behavioral, or both.54 More ominous signs of psychosis and and get revised with the advent of new technology or under-
confusion herald the dementia and myoclonus considered the standing, there remains more controversy than consensus.65
hallmarks of CJD. Patients generally die within a year, becom- Table 21-2 presents a simplified and useful framework for this
ing spastic, mute, and finally stuporous. Suggestive diagnostic discussion. Partial seizures have focal onset, but may second-
findings late in the clinical course include cerebellar atrophy arily generalize. They may, or may not, cause motor or auto-
on head computed tomography (CT) scan and typical EEG nomic signs, as well as somatosensory or psychic symptoms.
changes (Figure 21-1155). Magnetic resonance imaging (MRI) Generalized seizures include absence seizures (which may or
has also been found to be a reasonably sensitive and highly may not include motor signs), as well as the more readily
specific diagnostic aid,56 although finding 14-3-3 protein in recognized convulsive seizures.66
the CSF may be the most discriminating evidence for the
disease.57 Complex Partial Seizures
Kuru, which translates as to shiver with fear, was endemic
A highly underdiagnosed condition, partial seizures are
among Papua New Guinea highlanders of a particular tribe
responsible for most of the non-convulsive seizures experi-
who ate the brains of their dead (transumption). The inci-
enced by an estimated 60% of epileptics in the US.67 These
dence of kurua fatal, dementing, transmissible spongiform
seizures, largely (62%) of unknown etiology,68 often derive
encephalitis (TSE) with progressive extrapyramidal signs
from deep, limbic brain structures, commonly the temporal
declined along with the incidence of ritual cannibalism.58
lobe, where abnormal impulses do not transmit to the surface
electrodes of the EEG in up to 40% of patients.64,69 This is
further support for seizures remaining a clinical diagnosis,
Epilepsy inferred, but never dismissed, by EEG interpretation.
Epilepsy affects approximately 2 million Americans, and has Patients with epilepsy come to psychiatric attention because
a life-time prevalence of 3%.59 A prime example of the need they have a high prevalence of psychiatric symptoms that are

Descargado para JORGE LUIS SALAZAR GMEZ (jrg.lusaga@gmail.com) en Instituto Technologico Estudios Superiores Monterrey - Monterrey de ClinicalKey.es por Elsevier en mayo 03,
2017. Para uso personal exclusivamente. No se permiten otros usos sin autorizacin. Copyright 2017. Elsevier Inc. Todos los derechos reservados.
Mental Disorders Due to Another Medical Condition 213

21

Figure 21-11. Typical EEG patterns in CJD. EEG tracing demonstrates generalized periodic triphasic sharp wave complexes (PSWC), at a rate
of approximately 1 per second, typical findings late in the disease. (From Weiser HG, Schindler K, Zumsteg D. EEG in Creutzfeldt-Jakob disease,
Clin Neurophysiol 117:935951, 2006.)

Figure 21-12. Scalp electrode EEG recording during left temporal lobe seizure. (Courtesy of Sydney S. Cash, MD, PhD, Department of Neurol-
ogy, Massachusetts General Hospital and Harvard Medical School.)

Descargado para JORGE LUIS SALAZAR GMEZ (jrg.lusaga@gmail.com) en Instituto Technologico Estudios Superiores Monterrey - Monterrey de ClinicalKey.es por Elsevier en mayo 03,
2017. Para uso personal exclusivamente. No se permiten otros usos sin autorizacin. Copyright 2017. Elsevier Inc. Todos los derechos reservados.
214 PART VII Mental Disorders Due to Another Medical Condition

Figure 21-13. Simultaneous intracranial and surface EEG tracings. The top tracings (red) are taken from electrodes that have passed through
the foramen ovale, where the onset of seizure activity in the left temporal lobe appears earlier and more dramatically than it does on the scalp
electrode tracings. The surface tracings remain subtle until quite late in the seizure. (Courtesy of Sydney S. Cash, MD, PhD, Department of
Neurology, Massachusetts General Hospital and Harvard Medical School.)

TABLE 21-2 Classification of Seizure Disorders either event. Benzodiazepines may improve both conditions.
There are features, however, that aid in the differentiation.75
Onset Consciousness Although the fear of fainting is common in panic attacks, the
Term Focal General Unimpaired Impaired actual loss or alteration of consciousness is much more
Partial X
common in partial seizures. Mild visual or auditory distortions
Simple X X may occur during panic attacks, but true hallucinations (espe-
Complex X X cially olfactory or gustatory) are more suggestive of seizure
Generalized X X activity. Similarly, automatisms (chewing or lip-smacking
movements) and a confusional state after the episode strongly
support a diagnosis of seizure. Another helpful difference is
the vivid memory of the event in panic attacks, which leads
indistinguishable from those of primary mental disorders. to the fear of fear that is classic for panic disorder and pre-
Over half of epileptics experience depression, with higher rates disposes to agoraphobia and avoidance. In contrast, following
for patients with complex partial seizures and those with a seizure, patients often have lack of awareness or partial
seizure foci in the left hemisphere.70 In contrast, the incidence memory of the event, and rarely develop agoraphobia. The
of depression in matched medical and neurological control time course is generally more defined for panic, lasting 10 to
groups is only 30%. This implies that depression may be 20 minutes, whereas ictal panic may last less than 30 seconds.75
seizure-induced limbic dysfunction, and may also explain the Complex partial seizures tend to begin with cognitive (e.g.,
fivefold increase in the suicide rate for epileptic patients as dj vu, jamais vu, or forced thinking), affective (e.g., fear,
compared to those in the general public. Even when control- depression, or pleasure), or perceptual (e.g., illusions or olfac-
ling for psychiatric disease, demographic and socioeconomic tory or gustatory hallucinations) auras, then a brief cessation
factors, epileptic patients were found to have a threefold higher of activity, a minute or less of unresponsiveness and automa-
rate of suicide.71 That risk may be as much as 25 times higher tismic behavior, and, finally, a short (e.g., lasting seconds to
for patients with temporal lobe epilepsy (TLE),72 although this half an hour) period of decreased or lack of awareness.
has not been consistently replicated.73 Patients with partial Psychotic symptoms are seen 6 to 12 times as often in
seizures also experience more anxiety than those in the general epileptics than in the general population. These include, but
public and in patients with other types of seizures.63 In fact, are not limited to, the brief hallucinatory, affective, or cognitive
partial seizures bear many of the hallmarks of panic attacks, auras, which are themselves the result of abnormal electrical
and the two may be difficult to distinguish. Seizure-related impulses (i.e., seizure activity). Seizures may also be followed
panic may be peri-ictal (i.e., pre-ictal, ictal or post-ictal) or by post-ictal delirium of relatively short duration. However,
inter-ictal (i.e., between and independent of seizure activity). after years of epilepsy, some patients develop an episodic or
Panic attacks and seizure-related panic may occur out of the more chronic, unremitting psychosis with hallucinations,
blue, with hyperarousal, intense fear, perceptual distortion, paranoia, and a circumstantial thought pattern63 (rather than
and dissociative symptoms (e.g., depersonalization or dereali- the common schizophrenic formal thought disorders of tan-
zation).74 Hyperventilation, a common symptom of panic, gentiality, derailment, and thought-blocking). The epileptics
also lowers the seizure threshold, and may appear to initiate preserved affective warmth is in sharp contrast to the affective

Descargado para JORGE LUIS SALAZAR GMEZ (jrg.lusaga@gmail.com) en Instituto Technologico Estudios Superiores Monterrey - Monterrey de ClinicalKey.es por Elsevier en mayo 03,
2017. Para uso personal exclusivamente. No se permiten otros usos sin autorizacin. Copyright 2017. Elsevier Inc. Todos los derechos reservados.
Mental Disorders Due to Another Medical Condition 215

flattening more commonly seen in schizophrenia. This chronic atypical or recalcitrant to usual therapies, the lack of definitive
psychosis, thought to be caused by sub-ictal, temporal lobe confirmation of the clinical diagnosis of epilepsy should not 21
dysrhythmias, is often heralded by personality change and dissuade the psychiatrist from starting a trial of an appropriate
remits with relatively lower neuroleptic doses.76 Inter-ictal per- anti-seizure medication.
sonality traits commonly ascribed to TLE include obsessional-
ity, dependence, hyperreligiosity, hypergraphia, hyposexuality,
and humorlessness.77 However, such traits remain unsubstanti-
Nutritional Deficits
ated by research with structured, diagnostic instruments.78 Less Given the highly processed and fortified nature of the Ameri-
controversial is the recognized difficulty to disengage from the can diet, nutritional deficits are relatively rare in this country,
TLE patients viscous, sticky, conversational style. although they still occur in select populations. Table 21-3 sum-
There are many explanations for violence, but in the rare marizes the physical and neuropsychiatric manifestations of
event that it occurs during a seizure, it is never purposeful or the more common deficiency states.
organized, though occasionally reflexive or defensive.79 There The physical, psychiatric, neurological, and behavioral con-
is controversy about a syndrome of episodic dyscontrol being sequences of pellagra are the result of niacin (nicotinic acid)
more common in patients with early-onset temporal or frontal deficiency and deficiency of the niacin precursor, tryptophan.
lobe epilepsy. However, such eruptions of uncontrollable rage, When untreated, pellagra progresses to encephalopathy,
out of proportion to the minor infractions that set them off, peripheral neuropathy, diarrhea, and chronic wasting. Typical
are also more common in psychotic or traumatized patients, dermatological findings include angular stomatitis, glossitis,
and those intellectually, educationally, socioeconomically, and, less often, a scaly, erythematous rash in sun-exposed skin
and psychosocially challenged. Most likely, when perpetrated areas (Casals necklace, see Figure 21-14).80 However, the
by patients with epilepsy, such outbursts are the result of psy- initial, non-specific manifestations (e.g., insomnia, fatigue,
chopathology (primary or inter-ictal) or brain pathology (e.g., irritability, anxiety, and depressed mood) may be understand-
brain injury that might also be the cause of seizures). Ictal ably misattributed to depression.81 In the absence of niacin
events are, after all, the result of random firing, and not the repletion, these progress to more ominous mental slowing,
root of focused, violent acts. confusion, psychosis, and dementia. Leg weakness may follow
The neuropsychiatric presentation does not fully define the as part of a spastic spinal syndrome that includes hyper-
psychiatric challenge of epilepsy. The psychosocial conse- reflexia, clonus, and extensor plantar responses. Rare in the US
quences of living with a seizure disorder; the difficulty of since cereal products have been niacin fortified,82 pellagra still
substantiating partial seizures; the affective, cognitive, and occurs in certain groups (e.g., alcoholics,80 anorexics,81 vegetar-
physical side effects of common anti-seizure medications; and ians in less developed countries, refugee populations, and
the seizure threshold-lowering effect of common psychiatric recipients of bariatric surgery). Pellagrous delirium, with con-
medications are among the clinical issues. The co-morbidity fusion and psychosis, is more common than frank dementia,
of seizures and inter-ictal mood symptoms does not confirm and may be overlooked diagnostically in the setting of alcohol
the diagnosis of mood disorder due to epilepsy, nor does that withdrawal.80 Dementia is indicative of prolonged, severe defi-
diagnosis negate the need to treat the mood disorder directly. ciency, and it may not respond as quickly, or as completely, to
Thus, when mood or other psychiatric syndromes seem niacin replacement as do other features.

TABLE 21-3 Clinical Manifestations of Vitamin Deficiency States


Clinical Manifestations
Vitamin Deficiency State Physical Neuropsychiatric Population at Risk
Niacin* Pellagra Angular stomatitis, glossitis, Insomnia, fatigue, irritability, Alcoholics
(nicotinic rash in sun-exposed areas, anxiety, depressed mood; if
Refugees
acid) peripheral neuropathy, still untreated: mental slowing,
Vegetarians in less developed
diarrhea, chronic wasting, confusion, psychosis, dementia
countries
hyper-reflexia, clonus,
extensor plantar response
Thiamine Wernicke-Korsakoff Wernickes encephalopathy: Korsakoffs syndrome: retrograde Alcoholics (may be precipitated
(vitamin B1) syndrome vomiting, horizontal amnesia, poor concentration, in asymptomatic alcoholics
nystagmus, ophthalmoplegia, apathy, agitation, depressed when glucose is administered
fever, ataxia mood, confusion, confabulation before thiamine repletion)
Beriberi Cardiovascular (wet beriberi): As above: poor memory and Victims of famine, or extreme
tachycardia, high output, learning, apathy, confusion, poverty
peripheral dilation, edema, confabulation
biventricular failure
Neurological (dry beriberi): Patients with cancer, AIDS,
peripheral neuropathy, hyperemesis
weakness, deep tendon Patients on dialysis
reflexes, sensory neuropathy
Cobalamin Megaloblastic Position and vibratory sense, Apathy, irritability, depressed Malabsorption
(vitamin B12) anemia abnormal reflexes, sphincter mood or mood lability; if Pernicious anemia
dyscontrol, peripheral prolonged and untreated: S/p gastric bypass
numbness and paresthesias delirium (megaloblastic Inadequate intake
madness), hallucinations, Vegetarians
paranoia, worsening cognition
*And deficiency of the niacin precursor, tryptophan.
AIDS, Acquired immunodeficiency syndrome; S/p, status post.

Descargado para JORGE LUIS SALAZAR GMEZ (jrg.lusaga@gmail.com) en Instituto Technologico Estudios Superiores Monterrey - Monterrey de ClinicalKey.es por Elsevier en mayo 03,
2017. Para uso personal exclusivamente. No se permiten otros usos sin autorizacin. Copyright 2017. Elsevier Inc. Todos los derechos reservados.
216 PART VII Mental Disorders Due to Another Medical Condition

In famine or extreme poverty, thiamine (vitamin B1) defi- single-system involvement occurs, most commonly a blend of
ciency causes beriberi, but in the US, the alcoholism-associated cerebral, neuropathic, and cardiovascular signs and symptoms
Wernicke-Korsakoff syndrome is more common. A sign of occurs. As with niacin deficiency, the initial symptoms tend to
the twenty-first century may be the recent reports of etiologic be non-specific (e.g., poor concentration, apathy, mild agita-
starvation as a sequelae to bariatric surgery.8385 Prolonged tion, or depressed mood), but are followed by more disabling
malignancy86 and hyperemesis gravidarum87 have also (infre- signs (e.g., confusion, amnesia, or confabulation) of pro-
quently) caused the syndrome. Figure 21-1588 demonstrates longed, severe deficiency. Iatrogenic conversion of asympto-
the MRI findings associated with Wernickes encephalopathy. matic thiamine deficiency to Wernicke-Korsakoff syndrome
Factors such as total caloric intake, activity, and genetics89 has been reported when glucose is given before thiamine
seem to mediate the presentation, because most with malnu- repletion.90 However, an extensive recent literature review
trition or alcoholism do not exhibit symptoms. Although found no evidence above case report/series to support this
long-held medical adage.91 The authors conclude that pro-
longed glucose administration without thiamine repletion
may be a risk factor, but that glucose administration in a
hypoglycemic patient should not be delayed; thiamine may be
co-administered, or given soon after the start of glucose
replacement. The role of thiamine in glucose metabolism is
shown in Figure 21-16.92
Megaloblastic macrocytic anemia, the hallmark of vitamin
B12 (cobalamin) deficiency, is associated with neurodegenera-
tive changes of the central and peripheral nervous systems,
which manifest as decreased position and vibratory sensation,
reflex abnormalities, sphincter dyscontrol, and peripheral neu-
ropathy (e.g., numbness and paresthesias).93 The result of
malabsorption (e.g., from lack of intrinsic factor [pernicious
anemia, Figure 21-1794], following gastric surgery, nitrous
oxide, whippit abuse95) or inadequate intake (e.g., vegetar-
ian diet), the initial, non-specific symptoms may be psychiat-
ric (e.g., apathy, irritability, depression, or labile mood), rather
than neurological or hematological. If untreated, more pro-
longed disease may manifest as the less frequent delirium
syndrome (i.e., megaloblastic madness) of prominent hallu-
cinations, paranoia, and worsening cognition. Early in the
course, symptoms may be reversible with treatment.96 However,
more prolonged disease leads to demyelination, degenerative
changes, and ultimately cell death that precludes full resolu-
tion of neurological manifestations.

Metabolic Encephalopathy
Figure 21-14. Erythematous, scaly rash in sun-exposed areas caused Acute changes in mental status (e.g., disorientation, distur-
by pellagra (i.e., niacin deficiency); the involved neck region has been bances of affect, behavior, cognition, or level of conscious-
called Casals necklace, after Don Gaspar Casal who first described ness) are indicative of metabolic, rather than primary
pellagra in 1735. psychiatric, disturbances. Agitated delirium and marked

A B C D

Figure 21-15. Brain MRI demonstrating isolated T2/FLAIR (A) hyperintense signal changes and Gadolinium-enhancement (B, C and D)
of both mammillary bodies. Also note the anterior vermal atrophy, a common finding in patients with chronic excessive alcohol consumption.
(From Beh SC, Frohman TC, Frohman EM. Isolated mammillary body involvement on MRI in Wernickes encephalopathy. J Neurol Sci 334:
12, 2013.)

Descargado para JORGE LUIS SALAZAR GMEZ (jrg.lusaga@gmail.com) en Instituto Technologico Estudios Superiores Monterrey - Monterrey de ClinicalKey.es por Elsevier en mayo 03,
2017. Para uso personal exclusivamente. No se permiten otros usos sin autorizacin. Copyright 2017. Elsevier Inc. Todos los derechos reservados.
Mental Disorders Due to Another Medical Condition 217

Glycolysis
Glucose
21
Lactate
LDH
Pyruvate PDG  TPP

Acetyl coenzyme A

Citrate
Oxaloacetate
Fuel is completely
Citric oxidized and coupled
ATP = adenosine triphosphate
acid to phosphorylation in
LDH = lactate dehydrogenase -ketoglutarate
cycle the electron-transport
PDH = pyruvate dehydrogenase
TPP = thiamine pyrophosphate chain to synthesize
KGDH = alpha-ketoglutarate ATP and regenerate
Succinyl -KGDH  TPP reducing substances
dehydrogenase
coenzyme A
Metabolic steps blocked in
thiamine deficiency
Figure 21-16. The role of thiamine in aerobic metabolism. A glucose load administered in the absence of thiamine increases the metabolic
demand for thiamine, which may precipitate or worsen lactic acidosis and trigger fulminant Wernicke-Korsakoff syndrome or wet or dry beriberi.
(From Romanski S, McMahon M. Metabolic acidosis and thiamine deficiency, Mayo Clin Proc 74:259263, 1999.)

Figure 21-17. Peripheral blood smear in pernicious anemia. Smear Figure 21-18. Icteric sclera as seen in hepatic encephalopathy. Jaun-
demonstrates anisocytosis, macrocytosis, and one hypersegmented dice, or icterus, a yellowish discoloration from increased bile pigments
neutrophil. (From Page Green R. Macrocytic Anemia. In: Porwit A, in the serum, may be readily identified in the sclera while skin and
McCullough J, Erber WN. Blood and Bone Marrow Pathology, mucous membrane findings remain more subtle. (From the University
ed 2, 2011.) of Utah, Spencer S. Eccles Health Sciences Library. http://
library.med.utah.edu/WebPath/CINJHTML/CINJ049.html.)

fluctuations of consciousness are generally recognized, but


quiet delirium (e.g., withdrawal, mild memory or cognitive
impairment, passivity) or anxiety may be overlooked, consid- becomes confused, disoriented, amnestic, incoherent, somno-
ered to be the patients baseline, or erroneously ascribed to lent, and finally comatose.99 Focal neurological signs may be
a primary mental disorder. more common than previously appreciated. When recog-
nized, they appear to be more common in cirrhotic women,
Hepatic Encephalopathy clear with resolution of the encephalopathy, and are not prog-
nostically significant.100
Hepatocellular failure, whether acute, subacute, or chronic,
may lead to an encephalopathic syndrome that spans the
neuropsychiatric gamut from minimal personality change to
Renal Insufficiency
coma.97 The earliest signs of mild intellectual deficits often Bizarre visual hallucinations are commonly seen in the delir-
precede the classic findings of asterixis and jaundice (Figure ium of acute renal failure, but the neuropsychiatric symptoms
21-18),98 and may be so subtle that only those most familiar of chronic renal insufficiency (CRI) may be less extreme and
with the patient will notice, and the patients conversational more varied, ranging from mild cognitive slowing (e.g., diffi-
ability may even limit family awareness. Other early signs culties in concentration, problem-solving, calculation), to
include confusion, slowed cognition, poor concentration, severe cognitive impairment and lethargy.101 Although appro-
sleepwake reversal, poor personal grooming, and depressed, priate dialysis may reverse some of the cognitive deficits,
labile, or irritable mood. Episodic disorientation, rage, or dialysis itself is associated with cognitive slowing, memory
inappropriate behavior may precede the progressive decline in problems, and concentration problems (though rarely demen-
cognition, memory, speech, and consciousness as the patient tia since the elimination of aluminum from dialysis

Descargado para JORGE LUIS SALAZAR GMEZ (jrg.lusaga@gmail.com) en Instituto Technologico Estudios Superiores Monterrey - Monterrey de ClinicalKey.es por Elsevier en mayo 03,
2017. Para uso personal exclusivamente. No se permiten otros usos sin autorizacin. Copyright 2017. Elsevier Inc. Todos los derechos reservados.
218 PART VII Mental Disorders Due to Another Medical Condition

solutions). Depression is more common in both CRI and


patients undergoing dialysis,102 with the probable additive
effects of related neurotransmission and endocrine dysfunc-
tion (e.g., hyperparathyroidism101). Dialysis patients with
depression are twice as likely to be hospitalized or to die
within 12 months as those without depression.103 A
Uremic patients are also at risk for partial (and convulsive)
seizures that are often unrecognized, but associated with all
of the aberrations of affect, behavior, cognition, perception,
and consciousness discussed previously.

Hypoglycemic Encephalopathy
Disorientation, confusion, bizarre behavior, and hallucina- B
tions, manifestations of hypoglycemia of any cause, may be
heralded by apprehension or restlessness. There may be physi- Figure 21-19. Stigmata of Graves disease. (A) demonstrates the
cal signs (e.g., tachycardia or diaphoresis) or symptoms (e.g., goiter. (B) demonstrates the exophthalmos common in Graves disease.
hunger or nausea). Without treatment, patients progress to a
stuporous state, followed by coma.104 The hippocampus is
particularly sensitive to repeated hypoglycemic insult, which
may result in permanent amnesia.105 In critically ill patients, Thyroid Dysfunction
hypoglycemia (as well as hyperglycemia and fluctuations in
blood glucose) significantly aggravates the critical illness- Like depression, hypothyroidism is more prevalent (4:1)
induced neurocognitive dysfunction.106 in women. Mild (or subclinical) hypothyroidism shares
many of the same, non-specific signs and symptoms of depres-
Diabetic Ketoacidosis sion (e.g., fatigue, lethargy, weight gain, decreased appetite,
depressed mood, and slowed mental and motor activity).112114
Before the onset of the well-recognized three Ps (polyphagia, These features, along with cold intolerance, may be attributed
polydipsia, and polyuria), poorly controlled diabetic patients to depression, aging, dementia, or Parkinsons disease. Physi-
may experience vague, non-descript symptoms, such as leth- cal findings (e.g., thin and dry hair, dry skin, constipation,
argy or fatigue, followed by headache, nausea, and vomiting. stiffness, coarse voice, facial puffiness, carpal tunnel syndrome,
The elderly are at particular risk of cognitive dysfunction lateral eyebrow loss, hearing loss, and delayed relaxation
from prolonged delirium caused by osmotic fluid shifts. phase of deep tendon reflexes) aid in the diagnosis of more
Atypical neuroleptics, used to treat patients with mood or prolonged disease. Myxedema madness, a syndrome of hal-
psychotic disorders (or agitation from delirium or dementia lucinations and paranoia, is a manifestation of late disease.115
in elderly patients), increase the risk of insulin-resistance Depressed patients who are found to be hypothyroid may
and associated complications (e.g., dyslipidemias, metabolic require both thyroid hormone replacement and antidepres-
syndrome).107 sant treatment, as the depressive symptoms persist after
hormone replacement in about 10% of patients.
Acute Intermittent Porphyria Hyperthyroidism, on the other hand, may manifest much
like generalized anxiety or panic, before any of the more classic
This rare, autosomal dominant enzyme deficiency, with a pre-
signs of Graves disease appear (Figure 21-19).116 Patients may
dilection for women, interferes with heme biosynthesis and
complain of feeling anxious, or they may be labile, restless,
causes porphyrin rings to accumulate.108 Symptomatic disease,
and fidgety. They may have difficulties with memory, attention,
with an onset between ages 20 and 50, may manifest as the
planning, and productivity.117 Other common, non-specific
classic triad (i.e., episodic, acute, colicky abdominal pain,
features include palpitations, tachycardia, diaphoresis, irrita-
motor polyneuropathy, and psychosis), or it may cause purely
bility, tremulousness, insomnia, weakness, and fatigue. Thyro-
psychiatric symptoms (e.g., anxiety, insomnia, depression,
toxicosis psychosis has also been described as a primarily
mood lability, or psychosis).109 Some previously undiagnosed
affective psychosis with mania and depression equally repre-
chronic psychiatric patients have been found to have acute
sented.118 Despite voracious appetite, patients lose weight.
intermittent porphyria (AIP). AIP predisposes to seizures, from
In the elderly, however, this hyperactive state may be replaced
both the neurological effects of the disease and from electro-
by apathy, psychomotor retardation, loss of appetite, and
lyte disturbances (e.g. hyponatremia) caused by hypothalamic
depression.114,119 Such patients may also experience more
involvement and SIADH, as well as vomiting and diarrhea.110
prominent proximal muscle wasting, heart failure, or atrial
Meprobamate, sulfonamide antibiotics, ergot derivatives, and
arrhythmias.120
many anti-seizure medications promote porphyrin synthesis
that can promote attacks. Medicines known to be safe include
the phenothiazines, glucocorticoids, narcotic analgesics, peni-
Parathyroid Dysfunction
cillin derivatives, insulin, gabapentin, aspirin, acetaminophen, Despite the lack of conclusive correlation with absolute serum
and bromides. Besides drugs, alcohol, caloric restriction, and calcium levels, symptoms of parathyroid dysfunction are
gonadal steroids (endogenous or exogenous) are also known thought to be associated with disturbances in calcium, phos-
to precipitate episodes.111 phate, and bone metabolism.121 Primary hyperparathyroidism,
associated with hypercalcemia and more common in women
in their 50s and 60s, may be asymptomatic for years, and then
Endocrine Disorders manifest as non-specific symptoms (e.g., fatigue, abdominal
Because endocrine disorders are now recognized earlier in pain, mental slowing, attentional or memory problems,
their clinical course, the psychiatric manifestations are gener- depressed mood, anxiety, personality change, apathy, leth-
ally depression and anxiety, rather than delirium or dementia argy). More advanced disease has serious medical implications
associated with more advanced disease. (e.g., hypertension, fractures, peptic ulcers, pancreatitis, kidney

Descargado para JORGE LUIS SALAZAR GMEZ (jrg.lusaga@gmail.com) en Instituto Technologico Estudios Superiores Monterrey - Monterrey de ClinicalKey.es por Elsevier en mayo 03,
2017. Para uso personal exclusivamente. No se permiten otros usos sin autorizacin. Copyright 2017. Elsevier Inc. Todos los derechos reservados.
Mental Disorders Due to Another Medical Condition 219

stones) as well as possible delirium, coma and death.122 Para Red cheeks
thyroidectomy has been advocated for improved quality of life 21
in symptomatic patients, but remains controversial in those
without symptoms.122,123 Hypoparathyroidism, associated Moon
Fat pads
with a gradual decline in serum calcium, can also cause delir- face
(buffalo
ium or personality change.124 More acute calcemic depletion hump)
results in tetany.

Adrenal Dysfunction Thin Bruisability


skin ecchymoses
Adrenal insufficiency, or hypocortisolism, may be primary
(e.g., autoimmune Addisons disease) or secondary (e.g., from
prolonged exogenous glucocorticoids). The initial manifesta-
tion may be largely psychiatric (e.g., apathy, negativism, social
withdrawal, poverty of thought, fatigue, depressed mood, irri-
tability, and loss of appetite, interest, and enjoyment), and High
misattributed to depression.124 Nausea, vomiting, weakness, B.P.
hypotension, and hypoglycemia may develop, followed by
delirium and coma, if glucocorticoids are not administered.
Red
Even with appropriate replacement, however, residual psychi-
striae
atric symptoms may need more specific treatment, with careful
consideration not to worsen hypotension. Several factors
may contribute to the development and persistence of psychi-
atric symptoms (e.g., glucocorticoid deficit, the associated Thin arms
elevations of adrenocorticotropic hormone [ACTH] and and legs
corticotropin-releasing factor [CRF], dehydroepiandrosterone
[DHEA] deficiency,125 or the lack of normal diurnal and stress
modulation in glucocorticoid replacement). Pendulous
Hypercortisolism may also occur in several ways. It may be abdomen
ACTH dependent (e.g., hypersecretion of ACTH) either from
a pituitary adenoma (Cushings disease) or a non-pituitary
malignancy (Cushings syndrome). A less common cause is
ACTH-independent Cushings syndrome (e.g., adrenal hyper-
secretion of cortisol from hyperplasia or tumor). Despite the
classic cushingoid presentation (truncal obesity, peripheral Poor
wasting, hirsutism, moon facies, acne, and striae [Figure wound
21-20126,127]), diagnosis is frequently delayed because this healing
usually incomplete syndrome is often preceded by psychiatric
symptoms (e.g., anxiety, panic, depression, extreme irritability,
crying, pronounced suicidality, and, rarely, psychosis).128 Exog-
enous steroids may produce a comparable clinical picture.
Some patients on prolonged steroid therapy are (psychiatri-
cally) exquisitely sensitive to small dose alterations. High
doses of steroids may trigger mania in some patients, and when
such patients require episodic steroid treatment, they should
receive mood-stabilizing medication as prophylaxis.129,130

Pituitary Dysfunction Osteoporosis;


compressed (codfish)
Because of its regulatory role in multiple body systems, pitui- vertebrae
tary perturbations are associated with the full spectrum of
psychiatric symptoms. Decreased pituitary function (e.g., Figure 21-20. Cushings syndrome: clinical findings.
Sheehans syndrome: post-partum hemorrhagic pituitary
destruction) results in mental slowing and depressed or labile
The most common of the demyelinating disorders, multiple
mood. Increased pituitary function (e.g., from a pituitary
sclerosis (MS) has significant psychiatric symptomatology and
adenoma or a functioning tumor) may result in a variety of
occurs in 50 to 60 individuals per 100,000, and historically as
syndromes (e.g., Cushings syndrome from adrenal hyperpla-
many as 160 per 100,000 white American women raised in
sia), depending on the hormonal target or system affected.124
northern latitudes.131 However, the latitude differential appears
to be decreasing,132 while the female-to-male ratio is increas-
Demyelinating Disorders ing.133 Amyotrophic lateral sclerosis (ALS) is a distant second,
with an annual incidence of 1.6 per 100,000.134 Metachro-
Although relatively uncommon, disorders of myelination
matic leukodystrophy, adrenoleukodystrophy, gangliosidoses,
come to psychiatric attention not only because the neuropsy-
and SSPE are other, even less prevalent, disorders.
chiatric symptoms may precede physical findings but also
because the nature of the sensory and motor manifestations
(i.e., intermittent, subjective, and variable) may suggest psy-
Multiple Sclerosis
chiatric causation (e.g., somatization, anxiety, depression, or Of uncertain (probably genetic and environmental) etiology,
malingering). The non-specific initial symptoms may include MS is an episodic, multi-focal, inflammatory cause of
mild changes in personality, mood, behavior, or cognition. demyelination, more prevalent in women (female:male

Descargado para JORGE LUIS SALAZAR GMEZ (jrg.lusaga@gmail.com) en Instituto Technologico Estudios Superiores Monterrey - Monterrey de ClinicalKey.es por Elsevier en mayo 03,
2017. Para uso personal exclusivamente. No se permiten otros usos sin autorizacin. Copyright 2017. Elsevier Inc. Todos los derechos reservados.
220 PART VII Mental Disorders Due to Another Medical Condition

Figure 21-22. Spinal cord in a patient with ALS. Luxol-fast-blue


stain demonstrates lateral column degeneration with gliosis. (From the
WebPath educational program at http://library.med.utah.edu/
WebPath/CNSHTML/CNS105.html.)

Figure 21-21. Brain MRI of patient with MS. White spots are the
multiple T2-weighted hyperintensities characteristic of MS. (From
Michigan State University College of Osteopathic Medicine.)

incidence of 3.6:2.0) and in cold and temperate climates, with


a typical onset between ages 20 and 40. There is involvement
of the spinal cord, brainstem, cerebellum, cerebral hemi-
spheres, and optic nerves. Psychiatric symptoms are prevalent
(95%) in patients at all stages of MS, and such symptoms
correlate poorly with markers of disease severity (e.g., MRI
findings [Figure 21-21135], physical symptoms, and length of
illness). Psychiatric symptoms may not resolve with remission Figure 21-23. Metachromatic sulfatide in peripheral nerve. (From
of physical symptoms.136 Irritability tends to be the earliest Ellison D. etal. Neuropathology: A Reference Text of CNS Pathology,
symptom. ed 3. Edinburgh, Mosby Elsevier, 2013.)
Over three-quarters of patients will have episodes of
depressed mood,136 accompanied by a high risk of suicidality,
especially in the first year of diagnosis.137 Unlike primary
depression, there is no gender or age bias for depression in 3 to 5 years of symptom onset142), and fewer psychiatric mani-
MS patients. However, men with MS have a higher rate of festations (e.g., uncontrollable, or pseudobulbar, laughing and
suicide than do women. The newly diagnosed, and those crying from degeneration in the cortical bulbar projections to
diagnosed before age 30,138,139 also have an increased risk of the brainstem). Despite the common practice of prescribing
suicide. Other common neuropsychiatric symptoms include antidepressants, only about 11% of ALS patients meet criteria
agitation, anxiety, apathy, disinhibition, hallucinations, or for depression.143 However, as many as one-third of patients
delusions. Persistent euphoria, historically associated with with ALS may show frontotemporal cognitive deficits that
MS, occurs in less than 10% of patients, generally in the setting do not correlate with site of onset; these deficits have not
of diffuse cerebral disease. Although about 25% may experi- been shown to limit survival (when controlling for disease
ence some episode of mild mood elevation, this should not severity).144
be confused with mania, which is exceedingly rare. An incon-
gruous, upbeat nature, out of step with either the patients Lipid Storage Disorders
condition or previous personality style, is common, and may
be what was previously termed euphoria. Over 50% of MS These rare causes of demyelination are autosomal recessive,
patients experience some cognitive decline; approximately enzyme deficiency states that can occur in adulthood with
20% to 30% have more serious deficits. While memory prob- both psychiatric and neuromuscular manifestations. Meta-
lems are common, severe dementia is not.140 Steroids and chromatic leukodystrophy (Figure 21-23145), despite a swift,
interferon preparations used to ameliorate the effects of MS terminal course in infancy, may manifest with more insidious,
may also be iatrogenic causes of psychiatric symptoms. progressive, cognitive decline (e.g., forgetfulness, deterioration
of work or school performance) and with personality changes
Amyotrophic Lateral Sclerosis in adolescents or adults. Cerebellar signs (e.g., gait distur-
bances, masked facies, or strange postures) precede the even-
The second most common demyelinating disorder, and the tual (subcortical type) dementia and psychosis, after which
most common degenerative motor neuron disease (Figure patients become mute and bedridden.146 Adrenoleukodystro-
21-22141), ALS has a relentless course (i.e., death occurs within phy (an x-linked, recessive disorder of males) causes adrenal

Descargado para JORGE LUIS SALAZAR GMEZ (jrg.lusaga@gmail.com) en Instituto Technologico Estudios Superiores Monterrey - Monterrey de ClinicalKey.es por Elsevier en mayo 03,
2017. Para uso personal exclusivamente. No se permiten otros usos sin autorizacin. Copyright 2017. Elsevier Inc. Todos los derechos reservados.
Mental Disorders Due to Another Medical Condition 221

insufficiency and it may present with the psychiatric manifes-


tations of primary Addisons disease, or with asymmetric 21
myelopathic findings (e.g., homonymous hemianopsia and
hemiparesis), aphasia, or dementia. Becoming more symmet-
ric with spastic paraparesis or demyelinating polyneuropathy,
the disease is progressive and generally untreatable (except for
allogeneic hematopoietic cell transplantation, if provided at a
very early stage of brain involvement;147 glucocorticoids may
be used to treat the adrenal deficit).148 The adult form of
Tay-Sachs disease (a lysosomal storage disease known as a
gangliosidosis) may cause psychotic symptoms and seizures,
whereas the neuromuscular effects (e.g., clumsiness or weak-
ness from spinocerebellar and lower motor neuron involve-
ment) are still mild. The cranial nerves (and therefore vision)
are spared.149 However, cognition and intelligence are more
commonly affected than previously recognized.150

Mitochondrial Disease
These maternally-inherited disorders of mitochondrial meta-
bolic dysfunction usually present in children, but may become
apparent in adulthood with a wide range of multi-system
disease. Organs and tissues especially vulnerable are those Figure 21-24. MRI of MELAS. Axial FLAIR shows hyperintense foci
heavily reliant on aerobic metabolism (e.g., CNS [include in a young boy with MELAS that defy vascular territory. (From Saneto
visual and auditory pathways], heart, liver, kidneys, skeletal RP, Friedman SD, Shaw DW. Neuroimaging of mitochondrial disease,
muscles). Psychiatric symptoms are more prevalent among Mitochondrion 8:396413, 2008.)
those with mitochondrial disorders (e.g., depression, anxiety,
mania, psychosis, cognitive impairment).151 There is also evi-
dence for dysfunctional mitochondrial involvement in certain
psychiatric disorders (e.g., schizophrenia, bipolar disorder, frontal, prefrontal, or basal ganglia). Other likely risk factors
major depression), as well as neurodegenerative disorders.152 for depression or secondary mania include subcortical atrophy
The acronym MELAS (mitochrondrial myopathy, encepha- and a personal or family history of mood disorder. Previous
lopathy, lactic acidosis, and stroke-like episodes) refers to a stroke may also predispose to post-stroke depression. Besides
syndrome of stroke-like episodes, migraines, diabetes, sei- the contribution of functional and potential psychosocial
zures, sensorineuronal hearing loss, short stature, and cardio- losses, post-stroke depression is likely mediated by pro-
myopathy that usually presents in adolescence or early inflammatory factors and effects on neurotransmitters158 (just
adulthood, but with rare cases of onset after the age of 40.153 as the pro-inflammatory and neuroendocrine effects of depres-
MELAS has a relapsingremitting pattern with the hallmark sion are independent risk factors for stroke163). Untreated
transient stroke-like episodes. Serum and CSF lactate are com- post-stroke depression lasts about a year; minor depression is
monly elevated. Imaging reflects white matter involvement, as less predictable and may have a more protracted course.
well as characteristic, transient, cerebrocortical lesions that Depression is predictive of worse stroke recovery and it is
defy vascular territory (Figure 21-24).154 associated with greater mortality, not because of a correlation
with initial stroke severity, but because depression is associ-
ated with poor rehabilitative effort. Since post-stroke depres-
Cerebrovascular Disease sion responds to the usual antidepressant therapies, early
Stroke has decreased to the fourth leading cause of death in recognition and intervention are essential for optimal stroke
the US, behind heart disease, cancer, and chronic lower respi- outcome.158
ratory disease.155 Although stroke risk doubles with each Two other post-stroke syndromes give the appearance of
decade of adult age, the increasing incidence of vascular events mood disorders (or mood dysregulation), but they are actually
in the younger populations is a concerning trend.156,157 disorders of affective expression, rather than mood. Patients
Ischemic events account for about 80% of strokes,134 with with pseudobulbar palsy have outbursts of uncontrollable
atherosclerotic thrombosis and cerebral embolism responsi- laughter or crying (and lack less intense expression, such as
ble for about one-third of these. Hemorrhagic stroke is usually smiling) out of proportion to their emotional experience. This
the result of essential hypertension, with spontaneous aneu- affective incontinence (the effect of multiple lacunar infarcts
rysmal rupture and arteriovenous malformation relatively rare on the descending corticobulbar and frontopontine path-
etiologies. ways)164,165 is associated with dysarthria, dysphagia, and bifa-
By far the most common post-stroke psychiatric manifesta- cial weakness. The other syndrome, aprosodia, is the inability
tion, occurring in approximately 40% of patients,158 is depres- to affectively modulate speech and gestures (motor aprosodia)
sion (i.e., major or minor depression), either in the immediate or the inability to perceive the emotional content of others
post-stroke period (65%) or at about 6 months following speech or gestures (sensory aprosodia). Careful scrutiny is
stroke. Approximately 25% of patients experience post-stroke necessary to distinguish the affective blunting of aprosodia
anxiety, and three-quarters of those patients had co-morbid from true depression in post-stroke patients.166
depressive disorders, which suggests that post-stroke anxiety
in the absence of depression is rare.159 Mania, also rare, is
associated with right-sided lesions160 of the orbitofrontal,
Toxins
basotemporal, basal ganglia, and thalamic areas. There is sug- There is insufficient research to support the existence of envi-
gestive,161 but controversial,158,162 evidence for a correlation ronmental illness or multiple chemical sensitivity, but there
between depression and stroke location (left hemisphere are environmental toxins (e.g., carbon monoxide [CO] or lead)

Descargado para JORGE LUIS SALAZAR GMEZ (jrg.lusaga@gmail.com) en Instituto Technologico Estudios Superiores Monterrey - Monterrey de ClinicalKey.es por Elsevier en mayo 03,
2017. Para uso personal exclusivamente. No se permiten otros usos sin autorizacin. Copyright 2017. Elsevier Inc. Todos los derechos reservados.
222 PART VII Mental Disorders Due to Another Medical Condition

known to cause recognizable syndromes that may, nonethe- and mercury thermometers have been largely eliminated,
less, be misattributed to primary physical or mental disorders. mercury continues to pose a threat because of its availability
CO poisoning, from defective heating or exhaust ventilation, in folk medicines, botanical preparations, and breakable cap-
can manifest as a non-descript flu-like syndrome (e.g., malaise, sules (used by certain cultural or religious sects to sprinkle
cough, and nausea). Low-level exposure of a more chronic mercury in the home or car).175
nature causes depressive symptoms and cognitive decline. Drugs of any sort (e.g., prescribed, over-the-counter, herbal,
More severe poisoning may cause delayed neuropsychiatric or recreational), in therapeutic or overdose proportions, are
syndrome (DNS), characterized by a range of symptoms from potential toxins to consider whenever there are alterations in
subtle cognitive effects that self-resolve with time, to memory behavior, cognition, consciousness, or personality. (If drugs
dysfunction, visual problems, parkinsonism, confabulation, are causative, however, the diagnosis would be substance-
and hallucinations (in severe exposures, imaging demonstrates related disorder, covered in other chapters.)
atrophic changes in the basal ganglia and corpus callosum).167
Low-level lead exposure, not solely a concern of young chil-
dren (although children are particularly vulnerable to the neu-
Neoplasm
rotoxicity168), also manifests with non-descript psychiatric Every possible manifestation of CNS pathology can be pro-
symptoms suggestive of depression (e.g., fatigue, sleepiness, duced by unfettered, local or diffuse, neoplastic growth inside
depressed mood, and apathy). Adults and adolescents are at the rigid cranial vault. Tumors tend to be less symptomatic
risk for excessive lead exposure from environmental, recrea- than ischemic strokes affecting comparable brain volume. In
tional, and occupational sources.169 Besides the well-known adults, brain metastases of non-CNS malignancies are more
risk of lead-based paint, running or biking in heavily trafficked common than primary brain tumors.176 There may be clinical
areas, doing home repairs or remodeling, and even drinking features that hint at the location, type, and nature (i.e., meta-
from leaded crystal increases ones exposure. Artists of various static versus primary) of the tumor. Psychiatric manifestations,
crafts are at risk (e.g., stained glass, ceramic, and lead-figure however, may also be related to paraneoplastic syndromes of
artisans), as are art conservators. Those who use firearms for non-brain primary malignancies.177
work or recreation should monitor their lead levels. Gasoline, Approximately 50% to 60% of primary brain tumors are
solvents, and cleaning fluids are sources of organic lead expo- gliomas (Figure 21-25176), with gradual, pancortical dissemina-
sure, associated with nightmares, restlessness, and psychotic tion that results in equally diffuse symptomatology (e.g., cog-
symptoms. Extreme levels produce seizures and coma.170 There nitive decline). A similar, non-specific pattern occurs with
is evidence that cumulative lead exposure earlier in life may lymphoma or multiple metastases. In contrast, the extrinsic
be correlated with cognitive decline, worsening executive func- growth with localized brain compression of meningiomas
tion, and poor dexterity later in life.171,172 (Figure 21-26176) (approximately 25% of primary brain
Mercury is associated with two distinct syndromes of toxic- tumors) presents a more focal, progressive pattern. Intracra-
ity. If the exposure is from the organic form (e.g., methylated nial lesions can also cause seizures, either from random excita-
mercury from contaminated fish), neurological symptoms pre- tion or lack of the usual inhibitory control. Seizures are rarely
dominate (e.g., motor-sensory neuropathy, cerebellar ataxia, from subcortical lesions, and generally correlate with diffuse
slurred speech, paresthesias, and visual field defects), with less cortical infiltration or (even minor) compression from men-
dramatic psychiatric manifestations (e.g., depression, irritabil- ingiomas. Metastases are more likely than primary brain
ity, or mild dementia).173 Toxic inorganic mercury exposure, tumors to cause constitutional symptoms (e.g., fever, weight
however, has an initial psychiatric presentation (i.e., the Mad loss, or fatigue).
Hatter syndrome) of depression, irritability, and psychosis, Brain tumors in adults are associated with psychiatric
followed by less striking neurological findings (e.g., tremor, symptoms in 50% of patients, most of whom (80%) have
weakness, and headache).174 Although occupational exposure frontal or limbic tumors. Frontal tumors cause incontinence

A B
Figure 21-25. Malignant glioma (gliobastoma multiforme) in the right frontal lobe. Coronal T1-weighted magnetic resonance imaging scan with
gadolinium before (A) and after (B) surgery. (From Stevens GHJ. Brain Tumors: Meningiomas and Gliomas. In: Cleveland Clinic: Current Clinical
Medicine, ed 2, 2010, Saunders.)

Descargado para JORGE LUIS SALAZAR GMEZ (jrg.lusaga@gmail.com) en Instituto Technologico Estudios Superiores Monterrey - Monterrey de ClinicalKey.es por Elsevier en mayo 03,
2017. Para uso personal exclusivamente. No se permiten otros usos sin autorizacin. Copyright 2017. Elsevier Inc. Todos los derechos reservados.
Mental Disorders Due to Another Medical Condition 223

of bowel and bladder, as well as personality change and with small cell carcinoma of the lung. Tumor production of
depression. Tumors of the temporal lobe can cause a range of hormones, or hormone-like substances, can give rise to the 21
psychiatric symptoms (e.g., depression, personality change, syndrome of inappropriate antidiuretic hormone secretion
memory dysfunction, aphasia, or Korsakoff syndrome), and (SIADH), as well as to hypercortisolism, hypercalcemia, or
they are particularly prone to cause seizures,177 commonly hyperparathyroidism, and all of the concomitant neuropsy
associated with ictal or inter-ictal psychotic symptoms. Tumors chiatric manifestations. However, neurological syndromes of
in the upper brainstem are associated with akinetic mutism paraneoplastic origin are largely immune-mediated. Antibod-
(i.e., an alert but immobile state). ies may be directed against onconeural antigens (intra-cellular)
Another extrinsic cancer-related source of psychiatric symp- or cell-surface antigens. Paraneoplastic syndromes with
toms, paraneoplastic syndromes, may not be recognized when onconeural antigens are relatively rare (although their pres-
they occur months (or years) before other symptoms lead to ence is highly predictive of underlying neoplasm), with the
tumor detection.178 A variety of non-CNS tumors (e.g., breast, initial presentation of ataxia (from cerebellar degeneration),
uterus, ovary, testicle, kidney, thyroid, stomach, colon) can dysarthria and nystagmus, followed by limbic encephalitis or
cause these syndromes, but they are most frequently associated encephalomyelitis. However, paraneoplastic-type syndromes
involving cell-surface antigens (targeting neuronal receptors
[e.g., NMDA receptors], channels, or synaptic proteins) are
both more common and less predictive of underlying tumor.
Nonetheless, in certain populations, up to 30%50% of
patients with anti-NMDAR encephalitis will have underlying
cancer (e.g., ovarian teratoma, lymphoma).179,180 Limbic
encephalitis can present with psychiatric symptoms (e.g.,
depression, anxiety, irritability, personality changes, psychosis,
catatonia) as well as seizures, memory loss, cognitive changes,
decreased consciousness, hypoventilation and autonomic
instability. Clinical findings may include mild-to-moderate
lymphocytic pleocytosis, oligoclonal bands, mild protein ele-
vation, as well as specific antibodies in CSF and serum. EEG
abnormalities are generalized slowing, possible epileptic activ-
ity, as well as the classic extreme delta brush, as in anti-
NMDAR encephalitis (Figure 21-27).181 Although there may be
mesiotemporal enhancement in limbic encephalitis (and later
atrophy), normal imaging does not exclude the diagnosis.
Paraneoplastic neurologic syndromes to onconeural antigens
do not respond well to treatment, whereas encephalitis involv-
ing cell-surface antigens appear to respond more favorably to
immunotherapy and tumor removal.179
Colloid cysts are an example of benign pathology causing
symptoms (e.g., depression, mood lability, psychosis, person-
Figure 21-26. Meningioma. This coronal MRI with gadolinium dem- ality change, position-dependent or intermittent headache)
onstrates a large meningioma extending from the middle fossa into the because they take up space in a confined area and thereby
cavernous sinus, but with notable absence of edema in the adjacent compress adjacent structures. These lesions of the third ven-
brain tissue. (From Winn, HR. Youmans Neurological Surgery, ed 6, tricle press on diencephalic structures and may further increase
2011, Saunders) intracranial pressure by obstructing the ventricle.182

Figure 21-27. Extreme delta brush EEG pattern in anti-NMDAR encephalitis. EEG in a young woman with anti-NMDAR encephalitis demonstrates
generalized rhythmic delta frequency activity at 22.5Hz with superimposed rhythmic beta frequency activity. (From Schmitt SE, Pargeon K,
Frechette ES, etal. Extreme delta brush: a unique EEG pattern in adults with anti-NMDA receptor encephalitis, Neurology 79:10941100, 2012.)

Descargado para JORGE LUIS SALAZAR GMEZ (jrg.lusaga@gmail.com) en Instituto Technologico Estudios Superiores Monterrey - Monterrey de ClinicalKey.es por Elsevier en mayo 03,
2017. Para uso personal exclusivamente. No se permiten otros usos sin autorizacin. Copyright 2017. Elsevier Inc. Todos los derechos reservados.
224 PART VII Mental Disorders Due to Another Medical Condition

Degenerative Disorders
The association between movement and emotion is nowhere
more apparent than in the function and dysfunction of
the basal ganglia, mediated by limbic and cortical inputs,
and the shared neurotransmitter systems (i.e., dopamine,
-aminobutyric acid [GABA], serotonin, norepinephrine
[noradrenaline]). Degenerative disorders in this area span
motor, sensory, and psychiatric (e.g., depression, psychosis,
dementia) manifestations.183 Movement symptoms may fluc-
tuate in proportion to the severity of emotional distress.
Parkinsons disease affects at least 1% of the population
over age 60, and the prevalence may be on the rise.184 Caused
by degeneration primarily in the pars compacta of the sub-
stantia nigra, the well-defined syndrome of bradykinesia,
rigidity, tremor, and characteristic gait and posture disorders
are further complicated by depression in an estimated 40% Figure 21-28. Kayser-Fleischer rings in Wilsons disease. These gold
(4% to 70%) of patients,185 as well as by dementia in some or green-brown copper deposits surrounding the cornea are almost
(with a higher prevalence than in age-matched controls). The always present when patients have neuropsychiatric symptoms. (From
depression may be the result of dopamine, and to a lesser Sullivan CA, Chopdar A, Shun-Shin GA. Dense Kayser-Fleischer ring
extent norepinephrine, depletion. Antidepressants, such as in asymptomatic Wilsons disease (hepatolenticular degeneration), Br
SSRI and SNRIs, appear to be effective without worsening J Ophthalmol 86:114, 2002, Figure 1.)
motor symptoms.186 Psychosis, which can be part of the
disease, is worsened by anticholinergic and dopaminergic
medications used to treat it.185
The major pathology of Huntingtons chorea, an auto-
somal dominant disorder, includes striatal destruction, GABA
depletion, atrophy of the caudate and putamen, and mild
wasting of frontal and temporal lobes. With a prevalence of
10 per 100,000 population,187 it manifests most commonly in
the fourth or fifth decades, although the variable onset may
occur in childhood, associated with a more aggressive course
(with a survival time that is approximately half that of adults).
Psychiatric symptoms are prominent throughout the disease
progression, and they may precede the classic choreiform
movements.183,188 Initial signs may be significant attentional
deficits, poor judgment, and executive dysfunction, followed
by depression, apathy, social withdrawal, and poor personal
hygiene. In some individuals, the manifestation may be sug-
gestive of obsessive-compulsive disorder (OCD)189 or schizo-
phrenia. Patients tend to be irritable and impulsive.183,188,190
Although the depression benefits from antidepressant phar- Figure 21-29. Malar rash of systemic lupus erythematosus.
macotherapy, the cognitive decline (like the movement disor- (From On-line Archives of Rheumatology at www.archrheumatol
der) is relentless, and it progresses to dementia. .net/atlas/case68.html. Copyright 1996-2006 On-line Archives of
A genetic deficiency of ceruloplasmin affecting 1 to 2 per Rheumatology.)
100,000 population,191 Wilsons disease (also called hepato-
lenticular degeneration)183 is an autosomal recessive defect in
copper excretion, associated with copper deposition in the
liver, brain, cornea, and kidney. Rarely symptomatic before age excess copper; residual psychiatric symptoms, however, should
6, onset of symptoms frequently occurs in adolescence, but be treated with standard pharmacotherapies.
may manifest later in adulthood. About half of patients have
liver manifestations (e.g., hepatitis, parenchymal disease, or
cirrhosis). The other half primarily have neuropsychiatric Immune Diseases
symptoms, virtually always accompanied by the pathogno- Systemic lupus erythematosus (SLE) is an inflammatory,
monic Kayser-Fleischer rings191 (i.e., gold or green-brown autoimmune disease with multi-system involvement (Figure
copper deposits surrounding the cornea [Figure 21-28]).192 The 21-29193 shows the characteristic butterfly rash), unknown
lenticular nuclei and, to lesser extents, the pons, medulla, cause, variable symptoms and time course, inconclusive diag-
thalamus, cerebellum, and cerebral cortex, are the brain targets nostic studies, and female predilection (9:1),194 all of which
of copper toxicity. Despite common neurological features conspire to misattribute the non-specific manifestations to
(e.g., tremor, chorea, spasticity, rigidity, dysphagia, dysarthria), primary mental disorder (e.g., major depression or somatiza-
cognition is generally unaffected (although the dysarthria may tion). Approximately half of patients initially have depression,
be mistaken for intellectual disability). Patients with neuro- sleep disturbance, mood lability, mild cognitive dysfunction,
logical symptoms tend to have psychiatric symptoms as well, or psychosis. Onset of symptoms is usually between ages 20
but approximately 10% to 25% of patients have purely psychi- and 50. SLE may be treated with steroids, with possible wors-
atric symptoms.191 Bizarre, probably frontal behavior is most ening of psychiatric symptoms.195
common, but patients also have schizophrenia-like syndromes, Acquired immunodeficiency syndrome (AIDS) is discussed
as well as bipolar and more typical depressive syndromes. earlier in the chapter and in other chapters of this book (see
Psychiatric symptoms may respond to successful removal of Chapter 57).

Descargado para JORGE LUIS SALAZAR GMEZ (jrg.lusaga@gmail.com) en Instituto Technologico Estudios Superiores Monterrey - Monterrey de ClinicalKey.es por Elsevier en mayo 03,
2017. Para uso personal exclusivamente. No se permiten otros usos sin autorizacin. Copyright 2017. Elsevier Inc. Todos los derechos reservados.
Mental Disorders Due to Another Medical Condition 225

21

A B C

D E F
Figure 21-30. Radiographic documentation of the injuries sustained to a 2-year-old child. (A) Left subdural hematoma. (B) Temporoparietal
edema on MRI. (C) Deceased signal intensity on diffusion-weighted MRI. (D) Blood clot in subdural space on FLAIR. (E) Posterior radiograph of
chest. (F) MRI obtained 2 months post-injury showing multicystic encephalomalacia and bilateral subdural hygromas. (From Sieswerda-
Hoogendoorn T, Boos S, Spivack B, etal. Abusive head trauma Part II: radiological aspects, Eur J Pediatr 171:617623, 2012.)

Trauma Limbic structures (e.g., anterior temporal lobes and the inferior
frontal lobes) are at particular risk of damage from blunt
There are 1.7 million traumatic brain injuries (see Chapter 82) trauma, which may manifest as cognitive slowing (e.g., inatten-
yearly in the US,196 with the highest incidence in young men. tion, distractibility, memory difficulties, perseveration, poor
Falls are the leading cause, particularly in the elderly, followed planning), personality change (e.g., irritability or impulsivity),
by motor vehicle accidents, violence, and sports. Head injuries or mood disturbance (e.g., depression, lability, anxiety).201
due to combat is also receiving more attention.197 Abusive Vague somatic complaints (e.g., headache, dizziness, fatigue,
head trauma is the leading cause of traumatic death in infants, insomnia) may be attributed to depression. Patients also expe-
as well as the most common cause of childhood death from rience bothersome sensory symptoms (e.g., photophobia,
abuse.198 Figure 21-30199 depicts the serious brain injury sus- blurred vision, hyperacusis, tinnitus). Multiple head injuries,
tained in abusive head trauma to children. Despite the drama advanced age, and use of drugs or alcohol predispose to post-
of penetrating head injuries (e.g., gunshot wounds), the symp- concussive syndrome (PCS),202 the prolonged cognitive, emo-
toms are generally focal and related to the size and location tional, and somatic impairment following head trauma that
of directly involved brain tissue. caused significant cerebral concussion.203 There is also research
Closed head injury, on the other hand, is both much more evidence that victims of traumatic brain injury (TBI) have an
common and much more complex, as demonstrated in Figure increased prevalence of Axis I and II pathology diagnosable
21-31.200 Neither the diffuse presentation nor the prolonged decades after the initial insult.204 Because brain-injured patients
sequelae may correlate with the apparent extent of the initial tend to be exquisitely sensitive to psychotropic medication,
insult, possibly because of the several mechanisms of injury small (i.e., geriatric) doses should be used.
that may occur with blunt trauma. Brain contusion and neuro-
nal damage, frequently followed by bleeding and edema,
may be the result of direct impact, parenchymal stretching,
EVALUATION
acceleration/deceleration and shearing forces, and microscopic Although the DSM-5 diagnosis of mental disorder DTAMC
tears.201 The cognitive, emotional, and somatic symptoms of should always be on the short list of possibilities, a high
neurobehavioral dysfunction can last from months to years. degree of medical suspicion serves to guide the evaluation

Descargado para JORGE LUIS SALAZAR GMEZ (jrg.lusaga@gmail.com) en Instituto Technologico Estudios Superiores Monterrey - Monterrey de ClinicalKey.es por Elsevier en mayo 03,
2017. Para uso personal exclusivamente. No se permiten otros usos sin autorizacin. Copyright 2017. Elsevier Inc. Todos los derechos reservados.
226 PART VII Mental Disorders Due to Another Medical Condition

A B
Figure 21-31. (A) Right frontoparietal epidural hematoma w/ characteristic swirl sign involving laceration to middle meningeal artery on non-
contrast CT scan. (B) Multifocal T2 hyperintense foci reflective of traumatic axonal injury. (From Gean AD, Fischbein NJ. Head trauma, Neuroim-
aging Clin N Am 20:527556, 2010.)

process to include the findings required to make the diagnosis detail or clarification to support a medical diagnosis, but
or to rule it out. This should be the case for any thorough the initial screening examination is the same as in any
psychiatric evaluation. evaluation. Laboratory, imaging, or other specialized tests
should be focused to confirm or rule out suspected diagnoses
(e.g., a sleep-deprived EEG in suspected case of complex
History partial seizures). The overly inclusive approach to tests and
History should be obtained from the patient and, as appropri- procedures carries a greater risk of iatrogenic harm than
ate, any other pertinent sources (e.g., medical records, caregiv- warranted by the unlikely possibility of discovering unsus-
ers, current or past health or mental health care providers). pected disease. Brain imaging is recommended in the case of
Especially when the patient may be a questionable or limited sudden onset, focal signs, rapid progression, infectious disease,
historian, family, close friends, or partners may be valuable or trauma.
sources, preferably with the patients permission (or not, in
the case of emergent situations). The usual historical topics
(i.e., medical, psychiatric, family, and psychosocial) should be
TREATMENT CONSIDERATIONS/STRATEGIES
covered with sufficient detail to hone in on possible medical To know the etiology does not imply that the mental disorder
causes, and to correlate the onset of physical and psychiatric is normal in that situation. Despite the unalterable course
symptoms. The medical history includes not only past of some stable, chronic, or progressive medical conditions
and present illnesses, procedures, and all medications but also (e.g., previous stroke or toxic exposures, degenerative or demy-
questions about travel, possible exposures, recent or remote elinating diseases), the accompanying psychiatric symptoms
head injury, seizures, and use of substances (e.g., caffeine, should be aggressively treated. When it is possible to reverse,
tobacco, alcohol, recreational drugs). Non-prescription drugs, mitigate, or control the offending medical condition, however,
herbal or natural supplements, folk remedies, or unusual appropriate medical treatment should be initiated. After cor-
dietary practices may also be of concern. The psychiatric rection of the underlying medical condition, some mental
history should focus on distinguishing typical from atypical disorders will resolve, some may lessen, and others will con-
features of primary mental disorder (e.g., onset, course, tinue a course similar to the primary mental disorder. Even
response to treatment, past episodes). Family history of similar those psychiatric symptoms that will resolve with treatment of
symptoms, other psychiatric disorders, medical illnesses that the underlying medical condition (e.g., panic attacks with
run in the family or early, unexplained deaths may provide hyperthyroidism) deserve comfort measures (e.g., benzodi-
clues. The social history provides a wealth of information azepine therapy for panic symptoms) while waiting for
about the patients life (e.g., education, occupation or work medical resolution. Depression may lag behind the correction
history, and living situation) and possible risk factors (e.g., of hypothyroidism and may require a longer course of antide-
recreational activities, intimate contacts, sexuality, violence, pressant therapy. Some medical conditions and the associated
incarceration). psychiatric symptoms may need chronic treatment, as is often
the case with epilepsy and inter-ictal depression. Other con-
siderations in treating patients with mental disorders DTAMC
Examination include the psychosocial stress of living with an acute or
The Mental Status Examination (MSE) is the standard clinical chronic medical condition and pre-existing primary mental or
survey to assess consciousness, affect, behavior, cognition, personality disorders that may exacerbate the psychiatric
speech, thought, judgment, and insight. Clusters or patterns symptoms, interfere with the medical treatment, and generally
of findings, or deficits, may direct the examiner to seek more complicate or compromise the clinical outcome.

Descargado para JORGE LUIS SALAZAR GMEZ (jrg.lusaga@gmail.com) en Instituto Technologico Estudios Superiores Monterrey - Monterrey de ClinicalKey.es por Elsevier en mayo 03,
2017. Para uso personal exclusivamente. No se permiten otros usos sin autorizacin. Copyright 2017. Elsevier Inc. Todos los derechos reservados.
Mental Disorders Due to Another Medical Condition 227

CONCLUSIONS 62. Kanner AM, Schachter SC, Barry JJ, etal. Depression and epi-
lepsy: epidemiologic and neurobiologic perspectives that may 21
The DSM-5 diagnosis of mental disorder DTAMC should be explain their high comorbid occurrence. Epilepsy Behav 24:156
part of the differential diagnosis of every patient. Medical 168, 2012.
suspicion and a working knowledge of the general categories 75. Kanner AM. Ictal panic and interictal panic attacks: diagnostic
of medical conditions are important to making the diagnosis, and therapeutic principles. Neurol Clin 29:163175, 2011.
80. Oldham MA, Ivkovic A. Pellagrous encephalopathy presenting
because neuropsychiatric symptoms may precede, coincide as alcohol withdrawal delirium: a case series and literature
with, or lag behind the medical presentation. Selective tests review. Addict Sci Clin Pract 7:12, 2012.
and procedures should be used to confirm or rule out sus- 85. Becker DA, Ingala EE, Martinez-lage M, etal. Dry Beriberi and
pected diagnoses. Some mental disorders will resolve with Wernickes encephalopathy following gastric lap band surgery.
treatment of the underlying medical condition, some will J Clin Neurosci 19:10501052, 2012.
require temporary comfort measures, and others may need 91. Schabelman E, Kuo D. Glucose before thiamine for Wernicke
ongoing aggressive treatment. Regardless of the underlying encephalopathy: a literature review. J Emerg Med 42:488494,
medical cause or its course, psychiatric distress should be alle- 2012.
viated to the fullest extent possible. 97. Bajaj JS, Wade JB, Sanyal AJ. Spectrum of neurocognitive impair-
ment in cirrhosis: implications for the assessment of hepatic
encephalopathy. Hepatology 50:20142021, 2009.
Access the complete reference list and multiple choice questions 101. McQuillan R, Jassal SV. Neuropsychiatric complications of
(MCQs) online at https://expertconsult.inkling.com chronic kidney disease. Nat Rev Nephrol 6:471479, 2010.
106. Duning T, van den Heuvel I, Dickmann A, etal. Hypoglycemia
aggravates critical illness-induced neurocognitive dysfunction.
KEY REFERENCES Diabetes Care 33:639644, 2010.
14. Spudich S. HIV and neurocognitive dysfunction. Curr HIV/AIDS 130. Nishimura K, Omori M, Sato E, etal. Risperidone in the treat-
Rep 10:235243, 2013. ment of corticosteroid-induced mood disorders, manic/mixed
28. Feder HM Jr, Petersen BW, Robertson KL, etal. Rabies: Still a episodes, in systemic lupus erythematosus: a case series. Psycho-
uniformly fatal disease? Historical occurrence, epidemiological somatics 53:289293, 2012.
trends, and paradigm shifts. Curr Infect Dis Rep 14:408422, 132. Koch-Henriksen N, Sorensen PS. Why does the north-south gra-
2012. dient of incidence of multiple sclerosis seem to have disappeared
29. Hemachudha T, Ugolini G, Wacharapluesadee S, etal. Human on the northern hemisphere? J Neurol Sci 311:5863, 2011.
rabies: neuropathogenesis, diagnosis, and management. Lancet 139. Pompili M, Forte A, Palermo M, etal. Suicide risk in multiple
Neurol 12:498513, 2013. sclerosis: a systematic review of current literature. J Psychosom Res
33. Fallon BA, Vaccaro BJ, Romano M, etal. Lyme borreliosis: neu- 73:411417, 2012.
ropsychiatric aspects and neuropathology. Psychiatr Ann 36:120 151. Mancuso M, Orsucci D, Ienco EC, etal. Psychiatric involvement
128, 2006. in adult patients with mitochondrial disease. Neurol Sci 3471
39. Miklossy J. Chronic or late Lyme neuroborreliosis: analysis of 3474, 2013.
evidence compared to chronic or late neurosyphilis. Open Neurol 158. Robinson RG, Spalletta G. Poststroke depression: a review. Can
J 6(Suppl. 1M9):146157, 2012. J Psychiatry 55:341349, 2013.
45. Sea AC, White BL, Sparling PF. Novel Treponema pallidum sero- 163. Pan A, Sun Q, Okereke OI, etal. Depression and risk of stroke
logic tests: a paradigm shift in syphilis screening for the 21st morbidity and mortality: a meta-analysis and systematic review.
century. Clin Infect Dis 51:700708, 2010. JAMA 306:12411249, 2011.
46. Cho TA, Venna N. Management of acute, recurrent, and chronic 179. Leypoldt F, Wandinger KP. Paraneoplastic neurological syn-
meningitides in adults. Neurol Clin 28:10611088, 2010. dromes. Clin Exp Immunol 175:336348, 2014.
50. Gutierrez J, Issacson RS, Koppel BS. Subacute sclerosing panen- 186. Richard IH, McDermott MP, Kurlan R, etal. A randomized,
cephalitis: an update. Dev Med Child Neurol 52:901907, 2010. double-blind, placebo-controlled trial of antidepressants in
53. Brown P, Brandel JP, Sato T, etal. Iatrogenic Creutzfeldt-Jakob Parkinson disease. Neurology 78:12291236, 2012.
disease, final assessment. Emerging Infect Dis 18:901907, 2012.

Descargado para JORGE LUIS SALAZAR GMEZ (jrg.lusaga@gmail.com) en Instituto Technologico Estudios Superiores Monterrey - Monterrey de ClinicalKey.es por Elsevier en mayo 03,
2017. Para uso personal exclusivamente. No se permiten otros usos sin autorizacin. Copyright 2017. Elsevier Inc. Todos los derechos reservados.
Mental Disorders Due to Another Medical Condition 227.e1

31. Centers for Disease Control and Prevention: National Lyme


REFERENCES disease risk map. Available at: <www.cdc.gov/ncidod/dvbid/ 21
1. American Psychiatric Association. Diagnostic and statistical lyme/riskmap.htm>; [accessed 8/19/13].
manual of mental disorders, ed 4, Washington, DC, 1994, Ameri- 32. Hajek T, Paskova B, Janovska D, etal. Higher prevalence of
can Psychiatric Association, pp 165174. antibodies to Borrelia burgdorferi in psychiatric patients than in
2. American Psychiatric Association. Diagnostic and statistical healthy subjects. Am J Psychiatry 159:297301, 2002.
manual of mental disorders, ed 5, Arlington, VA, 2013, American 33. Fallon BA, Vaccaro BJ, Romano M, etal. Lyme borreliosis: neu-
Psychiatric Association. ropsychiatric aspects and neuropathology. Psychiatr Ann 36:120
3. <http://www.psychiatry.org/File%20Library/Practice/DSM/ 128, 2006.
DSM-5/Changes-from-DSM-IV-TRto-DSM-5.pdf>; [accessed on 34. Wormser GP. Early Lyme disease. N Engl J Med 354:27942801,
8/18/13]. 2006.
4. Kosten TR, OConnor PG. Management of drug and alcohol 35. Depietropaola DL, Powers JH, Gill JM, etal. Diagnosis of Lyme
withdrawal. N Engl J Med 348:17861795, 2003. disease. Am Fam Physician 72:297304, 2005.
5. Stannard L. Department of Medical Microbiology, University 36. Tager FA, Fallon BA. Psychiatric and cognitive features of Lyme
of Cape Town, <http://web.uct.ac.za/depts/mmi/stannard/herpes disease. Psychiatr Ann 31:173181, 2001.
.html>; [accessed on 8/19/13]. 37. Hasset AL, Radvanski DC, Buyske S, etal. Psychiatric comorbid-
6. Kramer AH. Viral encephalitis in the ICU. Crit Care Clin 29:621 ity and other psychological factors in patients with chronic
649, 2013. Lyme disease. Am J Med 122:843850, 2009.
7. Herpes encephalitis: T2-weighted MR. Available at: 38. Csallner G, Hofmann H, Hausteiner-Wiehle C. Patients with
<www.med.harvard.edu/AANLIB/cases/case25/mr1/012.html>. organically unexplained symptoms presenting to a borreliosis
8. Whitley RJ, Gnann JW. Viral encephalitis: familiar infections and clinic: clinical and psychobehavioral characteristics and quality
emerging pathogens. Lancet 359:507514, 2002. of life. Psychosomatics 54:359366, 2013.
9. Kennedy PGE, Chaudhuri A. Herpes simplex encephalitis. J 39. Miklossy J. Chronic or late Lyme neuroborreliosis: analysis of
Neurol Neurosurg Psychiatry 73:237238, 2002. evidence compared to chronic or late neurosyphilis. Open Neurol
10. Levitz RE. Herpes encephalitis: a review. Heart Lung 27:209212, J 6(Suppl. 1M9):146157, 2012.
1998. 40. Golden MR, Marra CM, Holmes KK. Update on syphilis: resur-
11. The big picture book of viruses: Retroviridae. Available at: gence of an old problem. JAMA 290:15101514, 2003.
<www.virology.net/Big_Virology/BVretro.html>. 41. Lukehart SA, Holmes KK. Syphilis. In Fauci AS, Braunwald E,
12. HIV. Available at: <www.aidsactioncoalition.org/images/hiv_ Isselbacher KJ, etal., editors: Harrisons principles of internal medi-
virus.gif>. cine, ed 14, New York, 1998, McGraw-Hill.
13. Antinori A, Arendt G, Becker JT, etal. Updated research nosology 42. Kodama K, Okada S, Komatsu N, etal. Relationship between
for HIV-associated neurocognitive disorders. Neurology 69:1789 MRI findings and prognosis for patients with general paresis.
1799, 2007. J Neuropsychiatry Clin Neurosci 12:246250, 2000.
14. Spudich S. HIV and neurocognitive dysfunction. Curr HIV/AIDS 43. Gliatto MF, Caroff SN. Neurosyphilis: a history and clinical
Rep 10:235243, 2013. review. Psychiatr Ann 31:153161, 2001.
15. Heaton RK, Clifford DB, Franklin DR, etal. HIV-associated neu- 44. Marra CM, Maxwell CL, Smith SL, etal. Cerebrospinal fluid
rocognitive disorders persist in the era of potent antiretroviral abnormalities in patients with syphilis: association with clinical
therapy: CHARTER Study. Neurology 75:20872096, 2010. and laboratory features. J Infect Dis 189:369376, 2004.
16. Reid S, Dwyer J. Insomnia in HIV infection: a systematic review 45. Sea AC, White BL, Sparling PF. Novel Treponema pallidum sero-
of prevalence. Psychosom Med 67:260269, 2005. logic tests: a paradigm shift in syphilis screening for the 21st
17. APA. Practice guideline for the treatment of patients with HIV/ century. Clin Infect Dis 51:700708, 2010.
AIDS. Am J Psychiatry 157S:162, 2000. 46. Cho TA, Venna N. Management of acute, recurrent, and chronic
18. Roy KF (in collaboration with the APA Commission on AIDS). meningitides in adults. Neurol Clin 28:10611088, 2010.
HIV fact sheet: HIV and anxiety, Washington, DC, 2002, American 47. Medscape: <http://emedicine.medscape.com/article/232915-
Psychiatric Association. differential>; [accessed on 9/21/2013].
19. Lyketsos CG, Treisman GJ. Mood disorders in HIV infection. 48. Garg RK. Subacute sclerosing panencephalitis. J Neurol 255:1861
Psychiatr Ann 31:4549, 2001. 1871, 2008.
20. Mijch AM, Judd FK, Lyketsos CG, etal. Secondary mania in 49. Garg RK. Subacute sclerosing panencephalitis. Postgrad Med J
patients with HIV infection: are antiretrovirals protective? J Neu- 78:6370, 2002.
ropsychiatry Clin Neurosci 11:475480, 1999. 50. Gutierrez J, Issacson RS, Koppel BS. Subacute sclerosing panen-
21. De Ronchi D, Faranca I, Forti P, etal. Development of acute cephalitis: an update. Dev Med Child Neurol 52:901907, 2010.
psychotic disorders and HIV-1 infection. Int J Psychiatry Med 51. ztrk A, Grses C, Baykan B, etal. Subacute sclerosing panen-
30:173183, 2000. cephalitis: clinical and magnetic resonance imaging evaluation
22. Paul RH, Cohen RA, Stern RA. Neurocognitive manifestations of of 36 patients. J Child Neurol 17:2529, 2002.
human immunodeficiency virus. CNS Spectr 7:860866, 2002. 52. Imran M, Mahmood S. An overview of human prion diseases.
23. AIDS Information Switzerland/BoehringerIngelheim Interna- Virol J 8:559567, 2011.
tional GmbH. Available at: <www.aids-info.ch/aas-e-fr.htm>. 53. Brown P, Brandel JP, Sato T, etal. Iatrogenic Creutzfeldt-Jakob
24. Martin L, Tummala R, Fernandez F. Psychiatric management of disease, final assessment. Emerging Infect Dis 18:901907, 2012.
HIV infection and AIDS. Psychiatr Ann 32:133140, 2002. 54. Irani DN. The neuropsychiatry of the transmissible spongiform
25. Messenger SL, Smith JS, Rupprecht CE. Emerging epidemiology encephalopathies (prion diseases). Psychiatr Ann 31:207214,
of bat-associated cryptic cases of rabies in humans in the United 2001.
States. Clin Infect Dis 35:738747, 2002. 55. Weiser HG, Schindler K, Zumsteg D. EEG in Creutsfeldt-Jakob
26. Blanton JD, Dyer J, McBrayer J, etal. Rabies surveillance in the disease. Clin Neurophysiol 117:935951, 2006.
United States during 2011. J Am Vet Med Assoc 241:712722, 56. Collie DA, Sellar RJ, Zeidler M, etal. MRI of Creutzfeldt-Jakob
2012. disease: imaging features and recommended MRI protocol. Clin
27. Srinivasan A, Burton EC, Kuehnert MJ, etal. Transmission of Radiol 56:726739, 2001.
rabies virus from an organ donor to four transplant recipients. 57. Muayqil T, Gronseth G, Camicioli R. Evidence-based guideline:
N Engl J Med 352:11031111, 2005. diagnostic accuracy of CSF 14-3-3 protein in sporadic Creutzfeldt-
28. Feder HM Jr, Petersen BW, Robertson KL, etal. Rabies: Still a Jakob disease: report of the guideline development subcommit-
uniformly fatal disease? Historical occurrence, epidemiological tee of the American Academy of Neurology. Neurology 79:
trends, and paradigm shifts. Curr Infect Dis Rep 14:408422, 2012. 14991506, 2012.
29. Hemachudha T, Ugolini G, Wacharapluesadee S, etal. Human 58. Alpers MP. The epidemiology of kuru: monitoring the epidemic
rabies: neuropathogenesis, diagnosis, and management. Lancet from its peak to its end. Philos Trans R Soc B 363:37073713,
Neurol 12:498513, 2013. 2008.
30. Haynes EB, Piesman J. How can we prevent Lyme disease? N Engl 59. Chang BS, Lowenstein DH. Epilepsy. N Engl J Med 349:1257
J Med 348:24242430, 2003. 1266, 2003.

Descargado para JORGE LUIS SALAZAR GMEZ (jrg.lusaga@gmail.com) en Instituto Technologico Estudios Superiores Monterrey - Monterrey de ClinicalKey.es por Elsevier en mayo 03,
2017. Para uso personal exclusivamente. No se permiten otros usos sin autorizacin. Copyright 2017. Elsevier Inc. Todos los derechos reservados.
227.e2 PART VII Mental Disorders Due to Another Medical Condition

60. Gross RA. A brief history of epilepsy and its therapy in the 88. Kaineg B, Hudgins PA. Images in clinical medicine: Wernickes
Western Hemisphere. Epilepsy Res 12:6574, 1992. encephalopathy. N Engl J Med 352:18e, 2005.
61. Riggs AJ, Riggs JE. Epilepsys role in the historical differentiation 89. Heap LC, Pratt OE, Ward RJ, etal. Individual susceptibility to
of religion, magic, and science. Epilepsia 46:452453, 2005. Wernicke-Korsakoff syndrome and alcohol-induced cognitive
62. Kanner AM, Schachter SC, Barry JJ, etal. Depression and epi- deficit: impaired thiamine utilization found in alcoholics and
lepsy: epidemiologic and neurobiologic perspectives that may alcohol abusers. Psychiatr Genet 12:217224, 2002.
explain their high comorbid occurrence. Epilepsy Behav 24:156 90. Koguchi K, Nakatsuji Y, Abe K, etal. Wernickes encephalopathy
168, 2012. after glucose infusion. Neurology 62:512, 2004.
63. Kanner AM. Recognition of the various expressions of anxiety, 91. Schabelman E, Kuo D. Glucose before thiamine for Wernicke
psychosis, and aggression in epilepsy. Epilepsia 45:2227, 2004. encephalopathy: a literature review. J Emerg Med 42:488494,
64. Bare MA, Burnstine TH, Fisher RS, etal. Electroencephalographic 2012.
changes during simple partial seizures. Epilepsia 35:715720, 92. Romanski S, McMahon M. Metabolic acidosis and thiamine defi-
1994. ciency. Mayo Clin Proc 74:259263, 1999.
65. Panayiotopoulos CP. The new ILAE report on terminology and 93. Berry N, Sagar R, Tripathi BM. Catatonia and other psychiatric
concepts for organization of epileptic seizures: a clinicians criti- symptoms with vitamin B12 deficiency. Acta Psychiatr Scand
cal view and contribution. Epilepsia 52:21552160, 2011. 108:156159, 2003.
66. Benbadis SR. Epileptic seizures and syndromes. Neurol Clin 94. Bain BJ. Diagnosis from the blood smear. N Engl J Med 353:498
19:251270, 2001. 507, 2005.
67. Browne TR, Holmes GL. Epilepsy. N Engl J Med 344:11451151, 95. Sethi NK, Mullin P, Torgovnick J, etal. Nitrous oxide whippit
2001. abuse presenting with cobalamin responsive psychosis. J Med
68. Banerjee PN, Hauser WA. Incidence and prevalence. In Engel J Toxicol 2:7174, 2006.
Jr, Pedley TA, editors: Epilepsy: a comprehensive textbook, ed 2, 96. Marsalha R, Rudoy I, Volkov I, etal. Symptomatic dietary
Baltimore, 2008, Wolters Kluwer/Lippincott Williams & Wilkins, vitamin B12 deficiency in a non-vegetarian population. Am J Med
pp 4556. 112:413416, 2002.
69. Blume WT, Holloway GM, Wiebe S. Temporal epileptogenesis: 97. Bajaj JS, Wade JB, Sanyal AJ. Spectrum of neurocognitive impair-
localizing value of scalp and subdural interictal and ictal EEG ment in cirrhosis: implications for the assessment of hepatic
data. Epilepsia 42:508514, 2001. encephalopathy. Hepatology 50:20142021, 2009.
70. Boylan LS, Flint LA, Labovitz DL, etal. Depression but not 98. Marquette University School of Dentistry, Oral and Maxillofacial
seizure frequency predicts quality of life in treatment-resistant Pathology. Available at: <www.dental.mu.edu//lesions/
epilepsy. Neurology 62:258261, 2004. yellow/yellow.htm>.
71. Christensen J, Vestergaard M, Mortensen PB, etal. Epilepsy and 99. Ferenci P, Lockwood A, Mullen K, etal. Hepatic encephalopathy
risk of suicide: a population-based case-control study. Lancet definition, nomenclature, diagnosis, quantification: final report
Neurol 6:693698, 2007. of the working party at the 11th World Congresses of Gastroen-
72. Harris EC, Barraclough B. Suicide as an outcome for mental terology, Vienna, 1998. Hepatology 35:716721, 2002.
disorders. A meta-analysis. Br J Psychiatry 170:205228, 1997. 100. Cadranel JF, Lebiez E, Di Martino V, etal. Focal neurologic signs
73. Nilsson L, Ahlbom A, Farahmand BY, etal. Risk factors for in hepatic encephalopathy in cirrhotic patients: an underesti-
suicide in epilepsy: a case control study. Epilepsia 43:644651, mated entity? Am J Gastroenterol 96:515518, 2001.
2002. 101. McQuillan R, Jassal SV. Neuropsychiatric complications of
74. Hurley RA, Fisher R, Taber KH. Sudden onset panic: epileptic chronic kidney disease. Nat Rev Nephrol 6:471479, 2010.
aura or panic disorder? J Neuropsychiatry Clin Neurosci 18:436 102. Fabrazzo M, de Santo RM. Depression in chronic kidney disease.
443, 2006. Semin Nephrol 26:5660, 2006.
75. Kanner AM. Ictal panic and interictal panic attacks: diagnostic 103. Hedayati SS, Bosworth HB, Briley LP, etal. Death or hospitaliza-
and therapeutic principles. Neurol Clin 29:163175, 2011. tion of patients on chronic hemodialysis is associated with a
76. Tadokoro Y, Oshima T, Kanemoto K. Interictal psychoses in physician-based diagnosis of depression. Kidney Int 74:930936,
comparison with schizophreniaa prospective study. Epilepsia 2008.
48:23452351, 2007. 104. Lo L, Tan CHA, Umapathi T. Diffusion-weighted MR imaging in
77. Blumer D. Evidence supporting the temporal lobe epilepsy per- early diagnosis and prognosis of hypoglycemia. Am J Neuroradiol
sonality syndrome. Neurology 53(5 Suppl. 2):S9S12, 1999. 27:12221224, 2006.
78. Devinsky O, Najjar S. Evidence against the existence of a tempo- 105. McNay EC, Cotero VE. Mini-review: Impact of recurrent hypogly-
ral lobe epilepsy personality syndrome. Neurology 53(5 Suppl. caemia on cognitive and brain function. Physiol Behav 100:234
2):S13S25, 1999. 238, 2010.
79. Tassinari CA, Tassi L, Calandra-Buonaura G, etal. Biting behav- 106. Duning T, van den Heuvel I, Dickmann A, etal. Hypoglycemia
ior, aggression, and seizures. Epilepsia 46:654663, 2005. aggravates critical illness-induced neurocognitive dysfunction.
80. Oldham MA, Ivkovic A. Pellagrous encephalopathy presenting Diabetes Care 33:639644, 2010.
as alcohol withdrawal delirium: a case series and literature 107. Llorente MD, Urrutia V. Diabetes, psychiatric disorders, and the
review. Addict Sci Clin Pract 7:12, 2012. metabolic effects of antipsychotic medications. Clin Diabetes
81. Park YK, Sempos CT, Barton CN, etal. Effectiveness of food 24:1824, 2006.
fortification in the United States: the case of pellagra. Am J Public 108. Puy H, Gouya L, Deybach JC. Porphyrias. Lancet 375:924937,
Health 90:727738, 2000. 2010.
82. Prousky JE. Pellagra may be a rare secondary complication of 109. Kumar B. Acute intermittent porphyria presenting solely with
anorexia nervosa: a systematic review of the literature. Altern Med psychosis: a case report and discussion. Psychosomatics 53:494
Rev 8:180185, 2003. 498, 2012.
83. Foster D, Falah M, Kadom N, etal. Wernicke encephalopathy 110. Solinas C, Vajda FJ. Neurological complications of porphyria.
after bariatric surgery: losing more than just weight. Neurology J Clin Neurosci 15:263268, 2008.
65:1987, 2005. 111. Anderson KE, Bloomer JR, Bonkovsky HL, etal. Recommenda-
84. Chaves LC, Faintuch J, Kahwage S, etal. A cluster of polyneu- tions for the diagnosis and treatment of the acute porphyrias.
ropathy and Wernicke-Korsakoff syndrome in a bariatric unit. Ann Intern Med 142:439, 2005.
Obes Surg 12:328334, 2002. 112. Razvi S, Ingoe LE, McMillan CV, etal. Health status in patients
85. Becker DA, Ingala EE, Martinez-lage M, etal. Dry Beriberi and with sub-clinical hypothyroidism. Eur J Endocrinol 152:713717,
Wernickes encephalopathy following gastric lap band surgery. 2005.
J Clin Neurosci 19:10501052, 2012. 113. Meier C, Staub JJ, Roth CB, etal. TSH-controlled L-thyroxine
86. Macleod AD. Wernickes encephalopathy and terminal cancer: therapy reduces cholesterol levels and clinical symptoms in sub-
case report. Palliat Med 14:217218, 2000. clinical hypothyroidism: a double blind, placebo-controlled trial
87. Spruill SC, Kuller JA. Hyperemesis gravidarum complicated by (Basel Thyroid Study). J Clin Endocrinol Metab 86:48604866,
Wernickes encephalopathy. Obstet Gynecol 99:875877, 2002. 2001.

Descargado para JORGE LUIS SALAZAR GMEZ (jrg.lusaga@gmail.com) en Instituto Technologico Estudios Superiores Monterrey - Monterrey de ClinicalKey.es por Elsevier en mayo 03,
2017. Para uso personal exclusivamente. No se permiten otros usos sin autorizacin. Copyright 2017. Elsevier Inc. Todos los derechos reservados.
Mental Disorders Due to Another Medical Condition 227.e3

114. Cooper DS, Biondi B. Subclinical thyroid disease. Lancet 142. Rowland LP, Schneider NA. Amyotrophic lateral sclerosis. N Engl
379:11421154, 2012. J Med 344:16881700, 2001. 21
115. Stowell CP, Barnhill JW. Acute mania in the setting of severe 143. Ganzini L, Johnston WS, Hoffman WF. Correlates of suffering in
hypothyroidism. Psychosomatics 46:259261, 2005. amyotrophic lateral sclerosis. Neurology 52:14341440, 1999.
116. Stigmata of Graves disease. Available at: <www.thachers.org/ 144. Rippon GA, Scarmeas N, Gordon PH, etal. An observational
internal_medicine.htm>. study of cognitive impairment in amyotrophic lateral sclerosis.
117. Stern RA, Robinson B, Thorner AR, etal. A survey of neuropsy- Arch Neurol 63:345352, 2006.
chiatric complaints in patients with Graves disease. J Neuropsy- 145. Metachromatic leukodystrophy. Available at: <http://155.37.5.
chiatry Clin Neurosci 8:181185, 1996. 42/eAtlas/CNS/1725.htm>.
118. Brownlie BE, Rae AM, Walshe JW, etal. Psychoses associated 146. Black DN, Taber KH, Hurley RA. Metachromatic leukodystrophy:
with thyrotoxicosisthyrotoxic psychosis. A report of 18 cases, a model for the study of psychosis. J Neuropsychiatry Clin Neurosci
with statistical analysis of incidence. Eur J Endocrinol 142:438 15:289293, 2003.
444, 2000. 147. Cartier N, Hacein-Bey-Abina S, Bartholomae CC, etal. Hemat-
119. Cooper DS. Hyperthyroidism. Lancet 362:459468, 2003. opoietic stem cell gene therapy with a lentiviral vector in x-linked
120. Mohandas R, Gupta KL. Managing thyroid dysfunction in the adrenoleukodystrophy. Science 326:818823, 2009.
elderly. Postgrad Med 113:5470, 2003. 148. Rosebush PI, Garside S, Levinson AJ, etal. The neuropsychiatry
121. Chiang CY, Andrewest DG, Anderson D, etal. A controlled, of adult-onset adrenoleukodystrophy. J Neuropsychiatry Clin Neu-
prospective study of neuropsychological outcomes post parathy- rosci 11:315327, 1999.
roidectomy in primary hyperparathyroid patients. Clin Endocri- 149. MacQueen GM, Rosebush PI, Mazurek MF. Neuropsychiatric
nol 62:99104, 2005. aspects of the adult variant of Tay-Sachs disease. J Neuropsychiatry
122. Roman S, Sosa JA. Psychiatric and cognitive aspects of primary Clin Neurosci 10:1019, 1998.
hyperparathyroidism. Curr Opin Oncol 19:15, 2007. 150. Frey LC, Ringel SP, Filley CM. The natural history of cognitive
123. Bilezikian JP, Khan AA, Potts JT. Guidelines for the management dysfunction in late-onset GM2 gangliosidosis. Arch Neurol
of asymptomatic primary hyperparathyroidism: summary state- 62:989994, 2005.
ment from the third international workshop. J Clin Endocrinol 151. Mancuso M, Orsucci D, Ienco EC, etal. Psychiatric involvement
Metab 94:335339, 2009. in adult patients with mitochondrial disease. Neurol Sci 3471
124. Huttu B. Subtle psychiatric presentations of endocrine diseases. 3474, 2013.
Psychiatr Clin North Am 21:905914, 1998. 152. Manji H, Kato T, Di Prospero NA, etal. Impaired mitochondrial
125. Hunt PJ, Gurnell EM, Huppert FA, etal. Improvement in mood function in psychiatric disorders. Nat Rev Neurosci 13:293307,
and fatigue after dehydroepiandrosterone replacement in Addi- 2012.
sons disease in a randomized, double blind trial. J Clin Endocri- 153. Hsu YC, Yang FC, Perng CL, etal. Adult-onset of mitochondrial
nol Metab 85:46504656, 2000. myopathy, encephalopathy, lactic acidosis, and stroke-like epi-
126. Biller BMK. Cushings syndrome. In Korenman SG, Molitch ME, sodes (MELAS) syndrome presenting as acute meningoencepha-
editors: Atlas of clinical endocrinology, vol IV, Neuroendocrinology litis: a case report. J Emerg Med 43:e163e166, 2012.
and pituitary disease, Philadelphia, 1999, Current Medicine. 154. Saneto RP, Friedman SD, Shaw DW. Neuroimaging of mitochon-
127. Cushingoid features. Available at: <www.meddean.luc.edu/ drial disease. Mitochondrion 8:396413, 2008.
lumen/meded/mech/cases/case16/list.htm>. 155. Towfighi A, Saver JL. Stroke declines from third to fourth leading
128. Geffken GR, Ward HE, Staab JP, etal. Psychiatric morbidity in cause of death in the United States: historical perspective and
endocrine disorders. Psychiatr Clin North Am 21:473489, 1998. challenges ahead. Stroke 42:23512355, 2011.
129. Wada K, Yamada N, Suzuki H, etal. Recurrent cases of 156. Kissela BM, Khoury JC, Alwell K, etal. Age at stroke: temporal
corticosteroid-induced mood disorder: clinical characteristics trends in stroke incidence in a large, biracial population. Neurol-
and treatment. J Clin Psychiatry 61:261267, 2000. ogy 79:17811787, 2012.
130. Nishimura K, Omori M, Sato E, etal. Risperidone in the treat- 157. Rosengren A, Giang KW, Lappas G, etal. Twenty-four-year trends
ment of corticosteroid-induced mood disorders, manic/mixed in the incidence of ischemic stroke in Sweden from 1987 to
episodes, in systemic lupus erythematosus: a case series. Psycho- 2010. Stroke 44:23882393, 2013.
somatics 53:289293, 2012. 158. Robinson RG, Spalletta G. Poststroke depression: a review. Can
131. Hernan MA, Olek MJ. Acherio A. Geographic variation of MS J Psychiatry 55:341349, 2013.
incidence in two prospective studies of US women. Neurology 159. Castillo CS, Schultz SK, Robinson RG. Clinical correlates of
53:17111718, 1999. early-onset and late-onset poststroke generalized anxiety. Am J
132. Koch-Henriksen N, Sorensen PS. Why does the north-south gra- Psychiatry 152:11741179, 1995.
dient of incidence of multiple sclerosis seem to have disappeared 160. Santos CO, Caeiro L, Ferro JM, etal. Mania and stroke: a system-
on the northern hemisphere? J Neurol Sci 311:5863, 2011. atic review. Cerebrovasc Dis 32:1121, 2011.
133. Alonso A, Hernn MA. Temporal trends in the incidence of mul- 161. Morris PLP, Robinson RG, Raphael B, etal. Lesion location and
tiple sclerosis: a systematic review. Neurology 71:129135, 2008. post-stroke depression. J Neuropsychiatry Clin Neurosci 8:399
134. Hirtz D, Thurman DJ, Gwinn-Hardy K, etal. How common 403, 1996.
are the common neurologic disorders? Neurology 68:326337, 162. Carson AJ, MacHale S, Allen K, etal. Depression after stroke and
2007. lesion location: a systematic review. Lancet 356:122126, 2000.
135. Brain MRI of patient with MS. Available at: <www.com.msu.edu/ 163. Pan A, Sun Q, Okereke OI, etal. Depression and risk of stroke
communique/2004_summer/06_ms.html>. morbidity and mortality: a meta-analysis and systematic review.
136. Diaz-Olavarrieta C, Cummings JL, Velazquez J, etal. Neuropsy- JAMA 306:12411249, 2011.
chiatric manifestations of multiple sclerosis. J Neuropsychiatry 164. Schiffer R, Pope LE. Review of pseudobulbar affect including a
Clin Neurosci 11:5157, 1999. novel and potential therapy. J Neuropsychiatry Clin Neurosci
137. Brnnum-Hansen H, Stenager E, Nylev Stenager E, etal. Suicide 17:447454, 2005.
among Danes with multiple sclerosis. J Neurol Neurosurg Psychia- 165. Miller A, Pratt H, Schiffer RB. Pseudobulbar affect: the spectrum
try 76:14571459, 2005. of clinical presentations, etiologies and treatments. Expert Rev
138. Scalfari A, Knappertz V, Cutter G, etal. Mortality in patients with Neurother 11:10771088, 2011.
multiple sclerosis. Neurology 81:184192, 2013. 166. Huffman J, Stern TA. Acute psychiatric manifestations of stroke:
139. Pompili M, Forte A, Palermo M, etal. Suicide risk in multiple a clinical case conference. Psychosomatics 44:6575, 2003.
sclerosis: a systematic review of current literature. J Psychosom Res 167. Guzman JA. Carbon monoxide poisoning. Crit Care Clin
73:411417, 2012. 28:537548, 2012.
140. Schiffer RB. Neuropsychiatric problems in patients with multiple 168. Chandran L, Cataldo R. Lead poisoning: basics and new devel-
sclerosis. Psychiatr Ann 32:128132, 2002. opments. Pediatr Rev 31:399405, 2010.
141. Lateral column degeneration in ALS. Available at: <http:// 169. Tong S, von Schirnding YE, Prapamontol T. Environmental lead
w w w. l i b r a r y. m e d . u t a h . e d u / W e b P a t h / C N S H T M L / exposure: a public health problem of global dimensions. Bull
CNS105.html>. World Health Organ 78:10681077, 2000.

Descargado para JORGE LUIS SALAZAR GMEZ (jrg.lusaga@gmail.com) en Instituto Technologico Estudios Superiores Monterrey - Monterrey de ClinicalKey.es por Elsevier en mayo 03,
2017. Para uso personal exclusivamente. No se permiten otros usos sin autorizacin. Copyright 2017. Elsevier Inc. Todos los derechos reservados.
227.e4 PART VII Mental Disorders Due to Another Medical Condition

170. Papanikolaou NC, Hatzidaki EG, Belivanis S, etal. Lead toxicity 188. Ranen NG. Psychiatric management of Huntingtons disease.
update. A brief review. Med Sci Monit 11:RA329RA336, 2005. Psychiatr Ann 32:105110, 2002.
171. Schwartz BS, Lee BK, Bandeen-Roche K, etal. Occupational lead 189. Beglinger LJ, Langbehn DR, Duff K, etal. Probability of obsessive
exposure and longitudinal decline in neurobehavioral test and compulsive symptoms in Huntingtons disease. Biol Psychia-
scores. Epidemiology 16:106113, 2005. try 61:415418, 2007.
172. Shih RA, Glass TA, Bandeen-Roche K, etal. Environmental lead 190. Paulsen JS, Ready RE, Hamilton JM, etal. Neuropsychiatric
exposure and cognitive function in community-dwelling older aspects of Huntingtons disease. J Neurol Neurosurg Psychiatry
adults. Neurology 67:15561562, 2006. 71:310314, 2001.
173. Wooltorton E. Facts on mercury and fish consumption. CMAJ 191. Lauterbach EC. Wilsons disease. Psychiatr Ann 32:114120,
167:897, 2002. 2002.
174. OCarroll RE, Masterton G, Dougall N, etal. The neuropsychiat- 192. Sullivan CA. Dense Kayser-Fleischer ring in asymptomatic
ric sequelae of mercury poisoning: the mad hatters disease revis- Wilsons disease (hepatocellular degeneration). Br J Ophthalmol
ited. Br J Psychiatry 167:9598, 1995. 86:114, 2002.
175. Saper RB, Phillips RS, Sehgal A, etal. Lead, mercury, and arsenic 193. Butterfly rash of SLE. Available at: <www.archrheumatol.net/
in US- and Indian-manufactured Ayurvedic medicines sold via atlas/case68.html>.
the Internet. JAMA 300:915923, 2008. 194. Helmick CG, Felson DT, Lawrence RC, etal. Estimates of the
176. DeAngelis LM. Brain tumors. N Engl J Med 344:114123, prevalence of arthritis and other rheumatic conditions in the
2001. United States, Part I. Arthritis Rheum 58:1525, 2008.
177. Phi JH, Kim S-K, Cho B-K, etal. Long-term surgical outcomes of 195. Ainiala H, Loukkola J, Peltola J, etal. The prevalence of neu-
temporal lobe epilepsy associated with low-grade brain tumors. ropsychiatric syndromes in systemic lupus erythematosus. Neu-
Cancer 115:57715779, 2009. rology 57:496500, 2001.
178. Darnell RB, Posner JB. Paraneoplastic syndromes involving the 196. American Psychiatric Association. Diagnostic and statistical
nervous system. N Engl J Med 349:15431554, 2003. manual of mental disorders, ed 5, Arlington, VA, 2013, American
179. Leypoldt F, Wandinger KP. Paraneoplastic neurological syn- Psychiatric Association, p 625.
dromes. Clin Exp Immunol 175:336348, 2014. 197. Rutland-Brown W, Langlois JA, Thomas KE, etal. Incidence of
180. Dalmau J, Gleichman AJ, Hughes EG, etal. Anti-NMDA-receptor traumatic brain injury in the United States, 2003. J Head Trauma
encephalitis: case series and analysis of the effects of antibodies. Rehabil 21:544548, 2006.
Lancet Neurol 7:10911098, 2008. 198. Piteau SJ, Ward MG, Barrowman NJ, etal. Clinical and radio-
181. Schmitt SE, Pargeon K, Frechette ES, etal. Extreme delta brush: graphic characteristics associated with abusive and nonabusive
a unique EEG pattern in adults with anti-NMDA receptor head trauma: a systematic review. Pediatrics 130:315323, 2012.
encephalitis. Neurology 79:10941100, 2012. 199. Sieswerda-Hoogendoorn T, Boos S, Spivack B, etal. Abusive
182. Aggleton JP, McMackin D, Carpenter K, etal. Differential cogni- head trauma Part II: radiological aspects. Eur J Pediatr 171:617
tive effects of colloid cysts in the third ventricle that spare or 623, 2012.
compromise the fornix. Brain 123:800815, 2000. 200. Gean AD, Fischbein NJ. Head trauma. Neuroimaging Clin N Am
183. Rosenblatt A, Leroi I. Neuropsychiatry of Huntingtons disease 20:527556, 2010.
and other basal ganglia disorders. Psychosomatics 41:2430, 201. Povlishock JT, Katz DI. Update of neuropathology and neuro-
2000. logical recovery after traumatic brain injury. J Head Trauma
184. Nussbaum RL, Ellis CE. Alzheimers disease and Parkinsons Rehabil 20:7694, 2005.
disease. N Engl J Med 348:13561364, 2003. 202. Jagoda A, Riggio S. Mild traumatic brain injury and the postcon-
185. Marsh L. Neuropsychiatric aspects of Parkinsons disease. Psycho- cussive syndrome. Emerg Med Clin North Am 18:355363, 2000.
somatics 41:1523, 2000. 203. American Psychiatric Association. Diagnostic and statistical
186. Richard IH, McDermott MP, Kurlan R, etal. A randomized, manual of mental disorders, ed 4, Washington, DC, 1994, Ameri-
double-blind, placebo-controlled trial of antidepressants in Par- can Psychiatric Association, pp 704705.
kinson disease. Neurology 78:12291236, 2012. 204. Koponen S, Taiminen T, Portin R, etal. Axis I and II psychiatric
187. Landles C, Bates GP. Huntington and the molecular pathogen- disorders after traumatic brain injury: a 30-year follow-up study.
esis of Huntingtons disease. EMBO Rep 5:958963, 2004. Am J Psychiatry 159:13151321, 2002.

Descargado para JORGE LUIS SALAZAR GMEZ (jrg.lusaga@gmail.com) en Instituto Technologico Estudios Superiores Monterrey - Monterrey de ClinicalKey.es por Elsevier en mayo 03,
2017. Para uso personal exclusivamente. No se permiten otros usos sin autorizacin. Copyright 2017. Elsevier Inc. Todos los derechos reservados.
Mental Disorders Due to Another Medical Condition 227.e5

MULTIPLE CHOICE QUESTIONS Lyme disease


Neurosyphilis
21
Select the appropriate answer.
Wernicke-Korsakoff syndrome
Q1 True or False. Mental Disorders Due to a Another
Medical Condition (DTAMC) are defined by the DSM-5 Q8 Which of the following conditions is MOST likely to be
as psychiatric conditions with symptoms severe enough manifest by neuropathy, ataxia, paresthesias, visual field
to merit treatment, and determined to be the direct, defects, depression, irritability, and psychosis?
physiological effect of a (non-psychiatric) medical
condition. Carbon monoxide poisoning
True Lead poisoning
False Lyme disease
Mercury toxicity
Q2 True or False. The onset of psychiatric symptoms that
coincides with the onset (or increased severity) of a Wernicke-Korsakoff syndrome
medical condition proves causation.
True
False

Q3 True or False. Psychiatric manifestations uncharacteristic MULTIPLE CHOICE ANSWERS


of primary mental disorders should raise the suspicion
Q1 The answer is: True.
of a direct physiological effect of a medical condition.
Mental Disorders Due to a Another Medical Condition
True
(DTAMC) are defined by the DSM-5 as psychiatric conditions
False with symptoms severe enough to merit treatment, and deter-
mined to be the direct, physiological effect of a (non-psychi-
Q4 Which of the following conditions is MOST likely to atric) medical condition. This conceptual language substitutes
have a predilection for the temporal and inferomedial for previously less useful, more dichotomous terms (e.g.,
frontal lobes and cause gustatory or olfactory organic versus functional) that minimized the psychosocial,
hallucinations or anosmia? environmental influences on physical symptoms, and implied
that psychiatric symptoms were without physiological cause.
Herpes simplex virus infection
General qualifiers further delineate Mental Disorders DTAMC
Lyme disease from other co-occurring medical and psychiatric conditions.
The mental disorder must be deemed the direct pathophysi-
Neurosyphilis
ological consequence of the medical condition, and must not
Rabies be better accounted for by another, primary mental disorder.
Symptoms that are only present during delirium (i.e., with
Thiamine deficiency
fluctuations in the level of consciousness and cognitive defi-
cits) are not considered DTAMC. Similarly, the presence of
Q5 In which of the following conditions are the pupils
dementia (e.g., with memory impairment with aphasia,
MOST likely to be small, irregular, unequal, able to
apraxia, agnosia, or disturbance of executive function) takes
accommodate, but not to react to light?
precedence over a diagnosis of DTAMC. Substance-induced
Herpes simplex virus infection symptoms (e.g., alcohol intoxication) do not meet criteria for
DTAMC.
Lyme disease
Neurosyphilis Q2 The answer is: False.
Rabies The determination of direct physiological causality is a
complex issue. The onset of psychiatric symptoms that coin-
Thiamine deficiency
cides with the onset (or increased severity) of the medical
condition is suggestive of, but does not prove, causation.
Q6 Which of the following conditions is MOST likely to be
Medical and mental disorders may merely co-exist. The initial
induced by the administration of glucose without prior
presentation of a medical condition may be psychiatric (e.g.,
thiamine repletion?
depression that manifests before the diagnostic awareness of
Herpes simplex virus infection pancreatic carcinoma), or the psychiatric symptoms may be
disproportionate to the medical severity (e.g., irritability in
Lyme disease
patients with negligible sensorimotor symptoms of multiple
Neurosyphilis sclerosis [MS]). It also happens that psychiatric symptoms may
occur long after the onset of medical illness (e.g., psychosis
Rabies
that develops after years of epilepsy). Psychiatric improvement
Wernicke-Korsakoff syndrome that coincides with treatment of the medical condition sup-
ports a causal relationship, although psychiatric symptoms
Q7 Which of the following conditions is MOST likely to be that do not clear with resolution of the medical condition do
manifest by psychosis, ataxia, myoclonus, and dementia not rule out causation (e.g., depression that persists beyond
and to be rapidly progressive and fatal? normalization of hypothyroidism). There are also Mental Dis-
orders DTAMC that respond to, and require, direct treatment
Creutzfeldt-Jakob disease
(e.g., inter-ictal depression), which should not be interpreted
Herpes simplex virus infection as evidence of a primary mental disorder.

Descargado para JORGE LUIS SALAZAR GMEZ (jrg.lusaga@gmail.com) en Instituto Technologico Estudios Superiores Monterrey - Monterrey de ClinicalKey.es por Elsevier en mayo 03,
2017. Para uso personal exclusivamente. No se permiten otros usos sin autorizacin. Copyright 2017. Elsevier Inc. Todos los derechos reservados.
227.e6 PART VII Mental Disorders Due to Another Medical Condition

Q3 The answer is: True. The signs and symptoms of neurosyphilis can be recalled with
the mnemonic PARESIS, and suggest the frontal and more
Psychiatric manifestations uncharacteristic of primary mental
diffuse nature of the syndrome. Personality change may be
disorders should raise the suspicion of a direct physiological
striking, and can involve apathy, poor judgment, lack of
effect of a medical condition. Features to consider include the
insight, irritability, and (new onset of) poor personal hygiene
age of onset (e.g., new-onset panic disorder in an elderly
and grooming. Patients may also have difficulty with calcula-
man), the usual time course (e.g., abrupt onset of depression),
tions and short-term memory. Later signs include mood labil-
and exaggerated or unusual features of related symptoms (e.g.,
ity, delusions of grandeur, hallucinations, disorientation, and
severe cognitive dysfunction with otherwise mild depressive
dementia.
symptoms). On the other hand, the typical manifestation of
a psychiatric syndrome supports the likelihood that medical In famine or extreme poverty, thiamine (vitamin B1) defi-
and mental disorders are co-morbid, but not causative. Such ciency presents as beriberi, but in the United States, the alco-
typical features include a history of similar episodes without holism-associated Wernicke-Korsakoff syndrome is the more
the co-occurrence of the medical condition, as well as a family common presentation. Although single-system involvement
history of the mental disorder. occurs, the most common presentation is a blend of cerebral,
neuropathic, and cardiovascular signs and symptoms. The
Q4 The answer is: Herpes simplex virus infection.
initial symptoms tend to be non-specific (e.g., poor concentra-
Herpes simplex virus (HSV) is the most frequent etiology of tion, apathy, mild agitation, or depressed mood), but are fol-
focal encephalopathy, and may cause either simple or complex lowed by more disabling signs (e.g., confusion, amnesia, or
partial seizures. With a predilection for the temporal and confabulation) of prolonged, severe deficiency.
inferomedial frontal lobes, HSV is well known to cause gusta-
tory or olfactory hallucinations, or anosmia (loss of the sense Q5 The answer is: Neurosyphilis.
of smell). This concentration in limbic structures may also
Historically, less than 10% of patients with untreated syphilis
explain the personality change, bizarre behavior, and psy-
develop a symptomatic form of parenchymatous neurosyphi-
chotic symptoms that some affected patients exhibit. Such
lis known as general paresis, 10 to 20 years after their initial
personality changes, cognitive difficulties, and affective lability
infection. Later signs include mood lability, delusions of gran-
may be persistent.
deur, hallucinations, disorientation, and dementia. It is during
Rabies is a viral infection of the central nervous system (CNS) this late stage that the classic neurological signs may appear
in mammals, generally transmitted by the infected saliva of an (e.g. tremor, dysarthria, hyperreflexia, hypotonia, ataxia, and
animal bite. The bite location, magnitude of the inoculum, Argyll Robertson pupils [small, irregular, unequal, able to
and extent of host defenses are the likely determinants of the accommodate, but not to react to light]). The diagnosis is
delay from contact to onset of symptoms, as the virus travels confirmed by cerebrospinal fluid (CSF) with elevated protein
along peripheral nerves centripetally to the CNS. Paresthesias and lymphocytes and a positive (CSF) Venereal Disease
or fasciculations at the bite location are characteristic aspects Research Laboratory (VDRL) slide test for treponemal
that distinguish rabies from viral syndromes with otherwise antibodies.
similar prodromes. Physical agitation and excitation give way
to episodic confusion, psychosis, and combativeness. These Q6 The answer is: Wernicke-Korsakoff syndrome.
episodes, possibly interspersed with lucid intervals, are the
In famine or extreme poverty, thiamine (vitamin B1) deficiency
harbinger of acute encephalitis, brainstem dysfunction, and
presents as beriberi, but in the United States, the alcoholism-
coma. Autonomic dysfunction, cranial nerve involvement,
associated Wernicke-Korsakoff syndrome is the more common
upper motor neuron weakness and paralysis, and often vocal
presentation. Factors (such as total caloric intake and activity)
cord paralysis occur. Roughly half of rabies infected humans
seem to mediate the presentation, as most with malnutrition
experience the classic hydrophobia (i.e., violent and severely
or alcoholism do not exhibit symptoms. Although single-
painful spasms of the diaphragm, laryngeal, pharyngeal, and
system involvement occurs, the most common presentation is
accessory respiratory muscles, triggered by attempts to swallow
a blend of cerebral, neuropathic, and cardiovascular signs and
liquids).
symptoms. As with niacin deficiency, the initial symptoms
The neuropsychiatric sequelae of Lyme disease require clini- tend to be non-specific (e.g., poor concentration, apathy, mild
cians in all areas to have a raised level of consciousness and agitation, or depressed mood), but are followed by more disa-
suspicion because the symptoms are non-specific, highly vari- bling signs (e.g., confusion, amnesia, or confabulation) of
able, often delayed, and recurrent. The target sites for the prolonged, severe deficiency. Iatrogenic conversion of asymp-
spirochete include the heart, eyes, joints, muscles, peripheral tomatic thiamine deficiency to Wernicke-Korsakoff syndrome
nervous system, or CNS, where it may lie dormant for so long may be induced by the administration of glucose without
(e.g., months to years) that memory of the initial bite has long prior thiamine repletion.
since faded.
Q7 The answer is: Creutzfeldt-Jakob disease.
Fatigue, irritability, confusion, labile mood, and disturbed
sleep may herald Lyme encephalitis. The much less common Creutzfeldt-Jakob disease (CJD) is a disease primarily of 50- to
presentation of Lyme encephalomyelitis may be confused with 70-year-olds. This rapidly progressive, fatal, prion disease is
multiple sclerosis. Some patients go on to develop chronic exceedingly rare, with most cases thought to be sporadic.
encephalopathy, a broad scope of persistent disturbances in Roughly 5% to 15% appear to be familial. Iatrogenic, person-
personality, behavior (e.g., disorganized, distractible, cata- to-person infection has also occurred following corneal trans-
tonic, mute, or violent), cognition (e.g., short-term memory, plantation, and therapeutic use of cadaveric human growth
memory retrieval, verbal fluency, concentration, attention, ori- hormone or cadaveric gonadotropins. The initial presentation
entation, and processing speed), mood (e.g., depressed, manic, may be non-specific and include problems of cognition
or labile), thought, or perception (e.g., paranoia, hallucina- (memory or judgment), mood (lability), perception (illusions
tions, depersonalization, hyperacusis, or photophobia). or distortions), or sensorimotor function (ataxic gait, vertigo,
Although extremely rare, more severe sequelae may include or dizziness). More ominous signs of psychosis and confusion
dementia, seizures, or stroke. herald the dementia and myoclonus considered the hallmarks

Descargado para JORGE LUIS SALAZAR GMEZ (jrg.lusaga@gmail.com) en Instituto Technologico Estudios Superiores Monterrey - Monterrey de ClinicalKey.es por Elsevier en mayo 03,
2017. Para uso personal exclusivamente. No se permiten otros usos sin autorizacin. Copyright 2017. Elsevier Inc. Todos los derechos reservados.
Mental Disorders Due to Another Medical Condition 227.e7

of CJD. Patients generally die within a year, becoming spastic, of a more chronic nature causes depressive symptoms and
mute, and finally stuporous. cognitive decline. More severe poisoning results in memory 21
dysfunction, visual problems, parkinsonism, confabulation,
Q8 The answer is: Mercury toxicity. psychosis, and delirium.
Mercury is associated with two distinct syndromes of toxicity. Low-level lead exposure, not solely a concern of young chil-
If the exposure is from the organic form (e.g., from contami- dren, also presents with non-descript psychiatric symptoms
nated fish), then neurological symptoms predominate (e.g., suggestive of depression (e.g., fatigue, sleepiness, depressed
motor-sensory neuropathy, cerebellar ataxia, slurred speech, mood, and apathy). Adults and adolescents are at risk for
paresthesias, and visual field defects), with less dramatic psy- excessive lead exposure from environmental, recreational, and
chiatric manifestations (e.g., depression, irritability, or mild occupational sources. Besides the well-known risk of lead-
dementia). Toxic inorganic mercury exposure, however, has an based paint, running or biking in heavily trafficked areas,
initial psychiatric presentation (i.e., the Mad Hatter syndrome) doing home repairs or re-modeling, and even drinking from
of depression, irritability, and psychosis, followed by less leaded crystal increases ones exposure. Artists of various crafts
striking neurological findings (e.g., tremor, weakness, and are at risk (e.g., stained glass, ceramic, and lead-figure arti-
headache). Although occupational exposure and mercury sans), as are art conservators. Those who use firearms for work
thermometers have been largely eliminated, mercury contin- or recreation should monitor their lead levels. Gasoline, sol-
ues to pose a threat because of its availability in folk medi- vents, and cleaning fluids are sources of organic lead exposure,
cines, botanical preparations, and breakable capsules (used by associated with nightmares, restlessness, and psychotic symp-
certain cultural or religious sects to sprinkle mercury in the toms. Extreme levels produce seizures and coma.
home or car).
Carbon monoxide poisoning, from defective heating or
exhaust ventilation, can present as a non-descript flu-like syn-
drome (e.g., malaise, cough, and nausea). Low-level exposure

Descargado para JORGE LUIS SALAZAR GMEZ (jrg.lusaga@gmail.com) en Instituto Technologico Estudios Superiores Monterrey - Monterrey de ClinicalKey.es por Elsevier en mayo 03,
2017. Para uso personal exclusivamente. No se permiten otros usos sin autorizacin. Copyright 2017. Elsevier Inc. Todos los derechos reservados.

You might also like